0% found this document useful (0 votes)
311 views150 pages

Benjamin Blackburn, Mumta Kanda, Lina Sprogyte - 180 MCQs For The Duke Elder Examination-Independently Published (2019)

Uploaded by

nicholaszotov
Copyright
© © All Rights Reserved
We take content rights seriously. If you suspect this is your content, claim it here.
Available Formats
Download as PDF, TXT or read online on Scribd
0% found this document useful (0 votes)
311 views150 pages

Benjamin Blackburn, Mumta Kanda, Lina Sprogyte - 180 MCQs For The Duke Elder Examination-Independently Published (2019)

Uploaded by

nicholaszotov
Copyright
© © All Rights Reserved
We take content rights seriously. If you suspect this is your content, claim it here.
Available Formats
Download as PDF, TXT or read online on Scribd
You are on page 1/ 150

180 MCQs for the Duke Elder Examination

Benjamin Blackburn
MBBS BSc (Hons)
4th in Duke Elder 2017

Mumta Kanda
MBBS BSc
2nd in Duke Elder 2018

Lina Sprogyte
MBBS BSc
2nd in Duke Elder 2017
Contents

Paper 1: 90 MCQs
Questions 2
Answers 40

Paper 2: 90 MCQs
Questions 73
Answers 107
© 2019 Benjamin Blackburn, Mumta Kanda and Lina Sprogyte
Benjamin Blackburn, Mumta Kanda and Lina Sprogyte have asserted
their right under the Copyright, Designs and Patents Act 1988 to be
identified as the authors of this work.
All rights reserved. No part of the publication may be reproduced,
stored in a retrieval system or transmitted, in any form or by any
means, electronic, mechanical, photocopying, recording or
otherwise, without prior permission of the copyright owner.
Preface
The Duke Elder Examination is notoriously challenging to prepare for
and sit for many medical students. There is a dearth of resources
and materials to help students prepare; in particular a lack of
practice questions reflecting the difficulty of the examination.
Practice examinations in particular are amongst the highest yield
strategies for studying and preparing for exams and due to the lack
of resources widely available to students we have prepared this MCQ
book to see to that particular need. The questions in this book are
all written by candidates who have scored amongst the top 5 in the
UK in previous years and have been designed to reflect the difficulty
and commonly tested topics in the examination.
The Duke Elder Examination format currently consists of 90 one in
four Multiple Choice Questions (MCQs) to be completed in two
hours. This book is structured to provide 2 mock examinations in
this format.
We hope you find this book useful in your preparation and wish you
all the best in the exam!
Benjamin Blackburn
Mumta Kanda
Paper 1

90 MCQs

2 hours
1) Which of the following medications is most likely to cause cystoid macular

oedema?

A. Glucocorticoids
B. NSAIDs
C. Epinephrine
D. Beta-blockers
E. Carbonic anhydrase inhibitors

2) A 9-year-old boy has neurofibromatosis type 1 (NF1). He is referred to


ophthalmology by his GP as he is complaining of worsening vision. Which of
the following ophthalmologic manifestations is not associated with NF1?

A. Optic nerve glioma


B. Eyelid (plexiform) neurofibroma
C. Absence of the greater wing of sphenoid
D. Myelinated nerve fibres
E. Lisch nodules
3) An 8-year-old child has severe blurring of vision in both eyes. On
examination the child has a saddle shaped nose deformity and notched,
small widely spaced teeth.
Given the likely diagnosis which layer of the cornea is most affected?

A. Epithelium
B. Bowman’s membrane
C. Stroma
D. Descemet’s membrane
E. Endothelium

4) You are in clinic and your consultant tells you that one of his patients had
a biopsy and the histological analysis revealed keratin pearls. Your consultant
asks you what condition this appearance is classically associated with.

A. Retinoblastoma
B. Squamous cell carcinoma
C. Basal cell carcinoma
D. Keratoacanthoma
E. Port wine stain
5) A 34-year-old HIV positive gentleman complains of visual impairment and
pain of both eyes. Examination of the cornea reveals a diffuse punctate
epithelial keratitis and biopsy reveals spores.
What is the most likely aetiological organism?

A. Toxoplasmosis
B. Microsporidium
C. Aspergillus
D. Candida
E. Herpes simplex

6) Which of the following is most likely to cause an oculomotor nerve palsy


without involvement of the pupil?

A. Berry aneurysm
B. Uncal herniation
C. Meningitis
D. Hypertension
E. Syphilis
7) Your consultant performs fundoscopy on a patient who was complaining of
headaches and visual disturbance. He tells you that he has seen multiple
capillary haemangiomas. The patient has also complained of difficulty with
balance and walking. What is the likely diagnosis?

A. Multiple Endocrine Neoplasia type 2A


B. Von Recklinghausen disease
C. Sturge Weber disease
D. Von Hippel Lindau disease
E. Usher syndrome

8) The nerve supplying the lacrimal gland synapses at which of the following
ganglia?

A. Ciliary ganglion
B. Pterygopalatine ganglion
C. Submandibular ganglion
D. Otic ganglion
E. None of the above

9) Which of the following is most likely to cause cicatrical entropion?

A. Onchocerciasis
B. Pseudomonal keratitis
C. Trachoma
D. Acanthamoeba keratitis
E. Candidal keratitis
10) Which of the following antibiotics is most effective against pseudomonas
infection?

A. Tobramycin
B. Chloramphenicol
C. Ethambutol
D. Moxifloxacin
E. Ertapenam

11) A 3-year-old child with Down syndrome is noted to have several grey
aggregations of connective tissue in his corneal stroma on examination of his
eyes. What is name given to this sign?

A. Fleischer ring
B. Brushfield spot
C. Lisch Nodule
D. Vogt’s limbal girdle
E. Becker’s sign

12) What is the commonest cause of blindness worldwide?

A. Cataract
B. Glaucoma
C. Trachoma
D. Refractive errors
E. Diabetes
13) Keyser Fleisher rings occur in Wilson disease due to copper deposition in
which structure?

A. Iris
B. Limbus
C. Descemet’s membrane
D. Corneal stroma
E. Bowman’s layer

14) A 45-year-old Hispanic lady comes to uveitis clinic for a follow-up after an
initial presentation several weeks ago. You notice that she has depigmented
patches on her hands and some hair of her eyelashes and eyebrows are
white. On examination her fundus has a characteristic ‘sunset glow’
appearance. Which of the following is she most likely to have?

A. Kearns-Sayre syndrome
B. Bardet-Bield syndrome
C. Vogt-Koyanagi-Harada syndrome
D. Usher syndrome
E. Gardner syndrome

15) A 68-year-old type II diabetic man complains of central visual blurring in


his right eye. He tells you that 6 weeks ago he underwent a surgical
procedure to remove a cataract in his right eye. What is the likely cause of
this man’s visual loss?

A. Diabetic retinopathy
B. Irvine-Gass syndrome
C. Endophthalmitis
D. Macular degeneration
E. Failure to remove cataract
16) A 12-year-old boy is noted to have a port wine stain over the right side of
his forehead that does not extend below the eye. He is known to have a
history of seizures. Given the likely diagnosis which of the following ocular
complications is he at most risk of?

A. Macular degeneration
B. Retinitis pigmentosa
C. Angle closure glaucoma
D. Optic atrophy
E. Secondary open angle glaucoma

17) A 5-year-old child with a squint is seen in clinic. On examination the


squint is present in all directions of gaze and the child can only fixate with
their left eye on a target. Covering up the left eye causes the right eye to
adduct as it takes up fixation. What is the likely diagnosis?

A. Esotropia
B. Esophoria
C. Exotropia
D. Exophoria
E. Hypotropia
18) A 29 year old woman complains of severe pain in her right eye that came
on suddenly this morning. She finds the pain is worse when looking at bright
lights. On examination of her eye you note mutton fat keratitic precipitates.
What is the most likely underlying aetiology of her condition?

A. Wegener’s granulomatosis
B. Sarcoidosis
C. Multiple sclerosis
D. Behcet’s disease
E. HIV

19) A previously healthy 61-year-old woman presents with a painful red right
eye. On further questioning she reveals that she has also been suffering from
joint pains. On examination of the eye you see a crescent shaped juxtalimbal
ulcerative lesion. Which of the following is most likely to account for her
symptoms?

A. Rheumatoid arthritis
B. Sarcoidosis
C. Multiple sclerosis
D. Behcet’s disease
E. HIV
20) A 40-year-old woman with a longstanding history of rheumatoid arthritis
complains of worsening vision and is especially troubled by glares from
various sources including car headlights. She finds she is having particular
difficulty with near vision rather than long distance vision. What is the most
likely cause of her glares?

A. Scleritis
B. Keratoconjunctivitis sicca
C. Chrysiasis
D. Posterior subcapsular opacity
E. Cortical cataract

21) Aqueous humour is secreted by which of the following structures?

A. Bruch’s membrane
B. Sclera
C. Choroid
D. Iris
E. Ciliary body
22) A 20-year-old male is noted to have yellowish slightly raised bumps on
the back of their neck and armpits. Your consultant asks you to perform
fundoscopy and asks you what you are most likely to see when examining
the retina.

A. Round oval lesion with elevated yellow subfoveal deposit


B. Oval detachment of sensory retina at the macula
C. Pale tessellated tigroid appearance of the retina
D. Parallel striae located at the posterior pole which are horizontally orientated
E. Dark red linear lesions with irregular serrated edges that intercommunicate in a
ring-like fashion around the optic disc

23) A 24-year-old man complains of blurred vision in his right eye. Direct
ophthalmoscopy reveals a well delineated oil droplet reflex and “scissoring” is
seen on retinoscopy. Given the likely diagnosis which of the following is also
a characteristic feature of this condition?

A. Fleischer rings
B. Mutton fat keratitic precipitates
C. Bull’s eye maculopathy
D. Angioid streaks
E. Sunset glow fundus
24) Which of the following conditions is characterized by cobblestone
papillae?

A. Trachoma
B. Keratoconjunctivitis sicca
C. Vernal conjunctivitis
D. Viral conjunctivitis
E. Staphylococcal conjunctivitis

25) Which of the following correctly describes part of the course of the
oculomotor nerve?

A. Originates at the third nerve nucleus at the level of the inferior colliculus
B. Passes between the superior cerebellar and posterior cerebral arteries
C. Pierces the dura mater anterior and lateral to the anterior clinoid process
passing between the borders of the tentorium cerebrii
D. Transverses the cavernous sinus within the medial segment
E. Divides into superior and inferior segments after entering the inferior orbital
fissure
26) Your consultant performs fundoscopy on a 30-year-old gentleman and
notes a “black sunburst retina lesion”. What condition is this finding
characteristic of?

A. Sickle cell anaemia


B. Ocular amyloidosis
C. Glaucoma
D. Macula degeneration
E. Diabetic retinopathy

27) Which condition is caused by a deficiency in fibrillin?

A. Ehlers-Danlos syndrome
B. Osteogenesis imperfecta
C. Marfan syndrome
D. Pseudoxanthoma elasticum
E. Posterior lenticonus
28) Which imaging modality is most useful for detecting thyroid eye disease?

A. MRI STIR sequence


B. MRI DWI sequence
C. CT orbits
D. Ultrasound
E. X-ray

29) What visual field defect is most likely to occur in a pituitary lesion?

A. Bitemporal hemianopia
B. Monocular blindness
C. Homonymous hemianopia
D. Superior quadrantanopia
E. Inferior quadrantanopia

30) A 64 year old man is due to have a cataract surgically removed. He has a
past medical history of BPH, asthma, hypertension and Parkinson’s disease.
He is taking omeprazole for gastro-oesophageal reflux. Which of the
following medications needs to be stopped pre-operatively to prevent
complications?

A. Tamsulosin
B. Salbutamol
C. Ropinirole
D. Amlodipine
E. Omeprazole
31) Which muscle is least commonly affected in thyroid ophthalmopathy?

A. Superior rectus
B. Inferior rectus
C. Lateral rectus
D. Medial rectus
E. Levator palpebrae superioris

32) Which of the following conditions is not identified as an immediate


priority in the framework of VISION 2020?

A. Cataract
B. Glaucoma
C. Refractive errors
D. Childhood blindness
E. Onchocerciasis

33) A patient with glaucoma has recently been started on medication to treat
his glaucoma. After starting the medication, he complains that his eyes have
become very red. His eyes are not painful and he has no visual disturbance.
Which of the following class of medications has he most likely been
prescribed?

A. Beta blockers
B. Prostaglandin analogue
C. Alpha-2 agonist
D. Topical carbonic anhydrase inhibitor
E. Muscarinic agonist
34) Which of the following conditions is most likely to be associated with
keratoconus?

A. Type II diabetes mellitus


B. Neurofibromatosis Type 1
C. Pseudoxanthoma elasticum
D. Leber congenital amaurosis
E. Polycystic kidney disease

35) A patient with acute angle closure glaucoma is started on acetazolamide.


Which of the following side effects is least associated with this medication?

A. Paraesthesia
B. Hyperkalaemia
C. Stevens-Johnson syndrome
D. Renal stones
E. Aplastic anaemia

36) Which aetiology of anterior uveitis is most associated with glaucoma?

A. Sarcoidosis
B. Wegener’s granulomatosis
C. Behcet disease
D. Ankylosing spondylitis
E. Fuch’s uveitis
37) What is the length of the intraosseous portion of the nasolacrimal duct?

A. 6mm
B. 8mm
C. 10mm
D. 12mm
E. 14mm

38) Concerning onchocerciasis which of the following is least appropriate?

A. It is treated with ivermectin


B. The vector is the Simulium blackfly
C. It may cause a pear-shaped pupil due to posterior synchiae
D. It can cause a punctate keratitis that progresses to sclerosing keratitis
E. Reproduction of microfilarae causes an intense inflammatory reaction which is
responsible for the clinical manifestations of disease

39) What is the definition of blindness using a LogMAR visual acuity scale?

A. Best corrected acuity < 0.00


B. Best corrected acuity <1.00
C. Best corrected acuity <-0.20
D. Best corrected acuity >1.00
E. Best corrected acuity >1.30
40) A 27-year-old woman with a past medical history of multiple sclerosis
notes experiencing double vision when looking to the left side. On
examination her eye movements are normal except on left gaze where her
right eye fails to adduct and her left eye shows nystagmus. Given the likely
cause of her diplopia what is the location of the lesion?

A. Right medial longitudinal fasciculus


B. Left medical longitudinal fasciculus
C. Right IIIrd nerve nucleus
D. Left IIIrd nerve nucleus
E. Pineal gland

41) Band keratopathy is commonly caused by deposition of which of the


following?

A. Magnesium salt
B. Potassium salt
C. Ferrous salt
D. Copper salt
E. Calcium salt

42) Which of the following organisms can penetrate an intact corneal


epithelium?

A. Streptococcus pyogenes
B. Staphylococcus aureus
C. Pseudomonas pyocyanaea
D. Corynebacterium diphtheriae
E. Pseudomonas aeruginosa
43) A Vossius ring is a deposition of pigment in which part of the eye?

A. Cornea
B. Lens
C. Anterior chamber
D. Sclera
E. Choroid

44) Which of the following is not a feature of Horner syndrome?

A. Miosis
B. Ptosis
C. Anhidrosis
D. True enophthalmos
E. Pseudo enophthalmos

45) A Marcus – Gunn pupil is caused by damage to which of the following


structures?

A. Cornea
B. Lens
C. Iris
D. Retina
E. Optic nerve

46) A colour-blind man has a mutation in X-chromosome resulting in a


complete lack of L-cones. What is his diagnosis?

A. Tritanopia
B. Deuteranomaly
C. Deuteranopia
D. Protanomaly
E. Protanopia
47) A 56-year-old woman comes to a glaucoma clinic for a follow-up. Both of
her parents had glaucoma and she would like to know more information
about the genetics of the disease. Which of the following genes are linked to
glaucoma?

A. TGFB1
B. MYOC & OPTN
C. ABCA4
D. CFH
E. RHO, RPGR, RP2 & RPE65

48) A 5-year-old girl is brought to A&E with a chemical burn to the eye. The
accident happened at home and her nanny immediately started washing the
eye with running tap water as advised by Google. Which of the following
features would indicate the most severe injury?

A. Pain severity is 10 out of 10


B. Injury with bleach
C. Corneal stroma is hazy and obscures iris details
D. Subconjunctival haemorrhages are present in all 4 quadrants of the globe
E. 8 clock hours of limbal ischaemia
49) An infant is examined using direct ophthalmoscopy and is found to have
a focal, white glistening bowl shaped excavation at the optic disc. What is the
most likely diagnosis?

A. Optic nerve hypoplasia


B. Morning glory anomaly
C. Optic disc coloboma
D. Myelinated nerve fibres
E. Tilted optic disc

50) Which of the following signs observed on clinical examination of the eye
is not caused by iron deposition?

A. Elschnig’s pearls
B. Hudson-Stahli’s line
C. Fleisher ring
D. Stocker’s line
E. Ferry’s line

51) A 54-year-old man comes to the Eye Casualty department due to severe
pain in the left eye. He works as a gardener in the Regent’s Park and tells
you that a few days ago he scratched his eye with plant matter. On
examination of the cornea, you note a grey stromal infiltrate with indistinct
fluffy margins and feathery extensions. Which of the following culture media
is most likely to grow the causative organism?

A. McConkey agar
B. Chocolate agar
C. Non-nutrient agar with E.coli
D. Sabouraud agar
E. McCoy media
52) An 83-year-old woman presents with visual deterioration in the left eye
following a left cataract surgery 2 months ago. She does not report any pain,
however she experiences floaters. Your consultant examines the lady and
tells you that he can see a white capsular plaque, which he thinks is caused
by Propionibacterium acnes. Which diagnosis is the consultant thinking
about?

A. Posterior capsular opacification


B. Phacoemulsification
C. Delayed-onset postoperative endophthalmitis
D. Anterior lenticonus
E. Sympathetic ophthalmitis

53) A patient is found to have an increased intraocular pressure (IOP) on a


routine examination. You decide to measure his central corneal thickness
(CCT) to get a better idea of the relevance of raised IOP. Which of the
following investigations are used to measure the CCT?

A. Perimetry
B. Specular microscopy
C. Tonometry
D. Biometry
E. Pachymetry
54) Which of the following genetic conditions has a mitochondrial inheritance
pattern?

A. Stargardt disease
B. Leber hereditary optic neuropathy
C. Best disease
D. Choroideremia
E. Stickler syndrome

55) A 65-year-old man with T2DM comes in for his annual diabetic
retinopathy screening. His fundus shows microaneurysms and hard
exudates, as well as a few dot-blot and flame-shaped haemorrhages and
cotton wool spots. Which of the following statements regarding the
pathophysiological processes in diabetic retinopathy is NOT correct?

A. Hard exudates form in the outer plexiform layer


B. Flame-shaped haemorrhages form in the retinal nerve fibre layer
C. Thickening of the capillary basement membrane is the first histological change
D. Cotton wool spots result from accumulation of axoplasmic material within the
nerve fibre layer
E. Dot-blot haemorrhages are found in middle retinal layers
56) Mrs Jones, a 78-year-old lady comes in to see her GP because she is
worried about her vision. Her mother went blind because of age-related
macular degeneration (AMD) and she has been recently diagnosed with very
mild non-exudative AMD. Mrs Jones is a non-smoker. She would like to know
how to decrease the risk of AMD progression. Which food supplements
should the GP recommend?

A. Vit C, vit E, Zn, Cu, lutein, zeaxanthin


B. Vit A, Zn, Cu, carotenoids
C. Vit A, lutein, zeaxanthin
D. Carotenoids, lutein, zeaxanthin
E. Zn, Cu, carotenoids, lutein

57) Collagens are important structural proteins in human body, including


ocular tissues. Sclera mainly consists of which collagen types?

A. Randomly arranged collagen type 2 and type 4


B. Regularly arranged collagen type 2 and type 4
C. Randomly arranged collagen type 1 and type 3
D. Regularly arranged collagen type 1 and type 3
E. All collagen types
58) A father is concerned about his son who will be three years old in a few
months. He thinks the child ‘cannot see well’. Given the boy’s age, which
visual acuity test would be most appropriate?

A. Keeler cards
B. Cardiff cards
C. Kay pictures
D. Hirschberg test
E. Modified logMAR

59) Regarding albinism, which of the following is NOT correct?

A. About 90% of optic nerve fibres cross to the contralateral side


B. Oculocutaneous albinism is more common than ocular albinism
C. Both boys and girls can be affected
D. Foveal hypoplasia is an important contributor to poor vision
E. Tyrosinase-positive albinism is a more severe form than tyrosinase-negative
albinism
60) Which of the following constitute a yoke muscle pair?

A. Left lateral rectus and left medial rectus


B. Right lateral rectus and left lateral rectus
C. Right inferior rectus and right superior oblique
D. Right superior rectus and left inferior oblique
E. Right inferior rectus and right inferior oblique

61) An elderly man with chronic anterior blepharitis comes to a clinic. He is


bothered by ongoing mild discomfort, redness and lacrimation. During a slit
lamp examination, you notice that his left cornea has marginal subepithelial
infiltrates separated from the limbus by a clear zone. Small epithelial defects
are also present. What is the most likely cause of these findings?

A. Disciform keratitis
B. Marginal keratitis
C. Terrien marginal degeneration
D. Pellucid marginal degeneration
E. Peripheral ulcerative keratitis associated with systemic autoimmune disease

62) In which city would you be most likely to meet a person with normal
intraocular pressure readings coming to a glaucoma clinic?

A. London
B. Tokyo
C. Shanghai
D. Abuja
E. Mumbai
63) You examine a right fundus of a HIV+ patient on a slit lamp using a
hand-held 90D lens. The fundus looks normal except for a single cotton wool
spot that you see in the inferior temporal quadrant. You know that lenses
used in conjunction with a slit lamp produce a virtual image of the fundus. In
which quadrant is the lesion in reality?

A. Superior nasal
B. Superior temporal
C. Inferior nasal
D. Inferior temporal

64) A 74-year-old lady complains of mild gradual onset visual disturbance.


However, on further questioning she tells you that although she is finding her
sight has slowly declined, she is no longer needing her prescription reading
glasses. Which of the following is most likely to have caused her visual
impairment?

A. Nuclear sclerosis
B. Macular degeneration
C. Posterior subcapsular cataract
D. Diabetic retinopathy
E. Cortical cataract

65) Given the same concentration of the drug, which one of the following is
the longest acting mydriatic?

A. Cyclopentolate
B. Tropicamide
C. Atropine
D. Homatropine
E. Pilocarpine
66) According to the DVLA, which patient would not be allowed to drive a
car?

A. A woman with cataracts and presbyopia whose vision improves to 6/12 with
glasses
B. A man suffering from right inferior homonymous quadrantanopia with macular
sparing, conserving 10 degrees of the central visual field on the right side
C. A man with prosthetic right eye and unaided 6/9 vision in the left eye
D. A colour blind person
E. A woman with visual fields 120 degrees horizontally that extend 60 degrees left
and 60 degrees right, with no significant defect in the binocular field

67) A student comes to a corneal clinic for a follow-up. He shows you his
new prescription from an optometrist: OD -3.00 -1.00 @ 175; OS -2.50 -1.50
@ 175. Which of the following is the correct conversion of this prescription
to a positive cylinder?

A. OD -3.00 +1.00 @ 085; OS -2.50 +1.50 @ 085


B. OD -4.00 +1.00 @ 085; OS -4.00 +1.50 @ 085
C. OD -3.00 +2.00 @ 085; OS -2.50 +1.00 @ 085
D. OD -2.50 +1.00 @ 045; OS -3.00 +1.50 @ 045
E. OD -4.00 +1.00 @ 045; OS -4.00 +1.50 @ 045
68) Electroretinogram (ERG) is a diagnostic test that measures electrical
activity generated by neuronal and non-neuronal cells in the retina in
response to a light stimulus. Which of the following statements is correct
regarding the ERG?

A. a wave reflects the depolarization of the photoreceptors due to closure of the


sodium ion channels
B. a wave is the initial corneal positive deflection
C. b wave is large and negative
D. c wave is derived from the RPE and photoreceptors
E. y wave predominantly results from Müller and ON-bipolar cells as these
depolarize in response to light

69) Human leukocyte antigen (HLA) system encodes cell-surface proteins


responsible for the regulation of the immune system in humans. Which of the
following HLA subtypes is strongly associated with Birdshot
retinochoroidopathy?

A. HLA-A29
B. HLA-B7
C. HLA-B27
D. HLA-B51
E. HLA-DR2
70) Degenerative (pathological) myopia is of great clinical significance as it
can be progressive, irreversible and affects individuals during their most
productive years. Which of the following features is least not a feature of this
condition?

A. Myopia more than -6D


B. Posterior staphyloma
C. Tilted optic disc
D. Choroidal folds
E. Lacquer cracks

71) Classically central retinal artery occlusion presents with sudden and
profound loss of vision. However there are cases when the visual acuity
might be normal. What anatomical structure is responsible for this
phenomenon?

A. Lamina cribrosa
B. Cilioretinal artery
C. Macular hole
D. Optic chiasm
E. Annulus of Zinn
72) You are examining a fundus of an elderly lady. You notice a dark
pigmented lesion at the posterior pole. Which feature would make you think
the lesion is a choroidal naevus rather than a malignant tumour?

A. The lesion is >5mm in diameter


B. The lesion is elevated > 1mm
C. Surface drusen are present
D. Orange surface pigment can be seen
E. Associated exudative retinal detachment

73) A 73-year-old man comes to see you complaining of some visual


disturbance. On examination of the optic disc you notice some mild disc
swelling bilaterally. Otherwise the exam is normal. His blood pressure is well
controlled and he is not complaining of any headaches. You suspect his
symptoms might be caused by one of his medications. Which of his drugs
would be most likely to cause optic neuropathy?

A. Allopurinol
B. Losartan
C. Chlorpromazine
D. Prednisolone
E. Amiodarone
74) Aqueous humour is produced by the ciliary body. From the posterior
chamber it moves though the pupil to enter the anterior chamber and exits
the eye via the trabecular and uveoscleral routes. Which statement about the
aqueous humour physiology is correct?

A. It is secreted by the pigmented epithelium of the ciliary processes


B. Trabecular route accounts for as much as 90% of outflow
C. It is produced at a rate of 10 ml/hr
D. Passive ultrafiltration is the main mechanism of aqueous production
E. The uveoscleral route is pressure sensitive

75) Which of the following are responsible for maintaining the blood-retinal
barrier (BRB)?

A. Non-fenestrated choriocapillaris and retinal astrocytes


B. Bruch’s membrane and tight junctions of retinal vascular endothelium
C. Bruch’s membrane and retinal podocytes
D. RPE tight junctions and non-fenestrated retinal vascular endothelium
E. RPE alone
76) Which law states that increased innervation to an extraocular muscle is
accompanied by a reciprocal decrease in innervation to its antagonist?

A. Snell’s law
B. Hering’s law
C. Sherrington’s law
D. Köllner’s rule
E. Listing’s law

77) Distichiasis is:

A. Posteriorly misdirected eyelashes


B. Partial or complete second row of eyelashes
C. Downward sagging of upper eyelashes
D. Excessive eyelash growth
E. Absence or decreased number of lashes

78) Which of the following is not a cause of leukocoria?

A. Eales disease
B. Toxocariasis
C. Coat’s disease
D. Retinopathy of prematurity
E. Hyperplastic primary vitreous
79) A 16-year-old girl comes to see you complaining of bilateral eyelid
swelling and a droopy left eyelid. She frequently experiences episodic
painless oedema of the upper eyelids that lasts a few days. On closer
inspection you also notice wrinkled atrophic lid skin. Which of the following is
a likely diagnosis?

A. Floppy eyelid syndrome


B. Epicanthus
C. Blepharophimosis
D. Blepharochalasis
E. Epiblepharon

80) Today you have seen 5 patients with rhegmatogenous retinal


detachments (RRD). Based on the information available below, which patient
should have a vitreoretinal surgery most urgently?

A. A patient with a U-tear in the superior temporal quadrant causing a progressive


peripheral visual field defect but normal central vision. His symptoms started
24 hours ago.
B. A patient with a U-tear in the superior nasal quadrant causing a peripheral
visual field defect and loss of central vision. His symptoms started 42 hours
ago.
C. A patient with an operculated tear in the superior temporal quadrant that
shows pigmentation around the break and intraretinal cysts with subretinal
demarcation lines.
D. A patient with a small U-tear in the inferior temporal quadrant and ‘tobacco
dust’ seen on examination.
E. A patient with a hole in the inferior nasal quadrant denying any visual
symptoms.
81) An 81-year-old woman is complaining of a missing central patch in the
visual field of her left eye. This has been bothering her for a few months
already. During examination, when a narrow-slit beam is projected over the
centre of the macula the beam appears thinned and broken. You order OCT,
which confirms the diagnosis. You decide that the patient will benefit from
treatment. Which treatment will you offer?

A. Intraocular anti-VEGF injection


B. Steroid eyedrops
C. Oral antibiotics
D. Panretinal photocoagulation
E. Pars plana vitrectomy

82) During retinal development, which of the following cells are formed the
earliest?

A. Bipolar cells
B. Retinal pigment epithelial cells
C. Rods
D. Cones
E. Müller cells
83) In retinal detachment, what detaches from what?

A. The Bruch’s membrane detaches from the choroid


B. The RPE detaches from the Bruch’s membrane
C. The photoreceptor layer detaches from the RPE
D. The bipolar cell layer detaches from the photoreceptor layer
E. The ganglion cell layer detaches from the bipolar cell layer

84) Which of the following anatomical terms describe the junction between
the retina and the ciliary body?

A. Pars plana
B. Schwalbe’s line
C. Ora serrata
D. Lamina cribrosa
E. Grey line

85) What is the embryonic origin of the lens?

A. Endoderm
B. Mesoderm
C. Neural crest cells
D. Neural ectoderm
E. Surface ectoderm
86) Evisceration is a surgical procedure that:

A. is done when a patient has an intraocular tumour


B. is done when a patient has orbital malignancy
C. removes the entire eyeball but allows orbital implants post-surgery
D. removes the cornea and the contents of the globe, leaving the sclera and
extraocular muscles intact
E. removes the globe and the soft tissues of the orbit

87) Trachoma is the leading infectious cause of blindness globally. The WHO
developed a strategy known by the acronym ‘SAFE’ that aims to eliminate
trachoma as a blinding disease by 2020. What are the 4 interventions of the
‘SAFE’ strategy?

A. Surgery + Antibiotics + Financial support + Elimination of mosquitos


B. Surgery + Antibiotics + Facial hygiene + Environmental improvement
C. Surgery + Access to clean water + Financial support + Elimination of flies
D. Sewage improvement + Access to clean water + Foreign doctors + Education
E. Sewage improvement + Antibiotics + Foreign doctors + Education
88) Which of the following ophthalmological conditions is NOT associated
with rheumatoid arthritis?

A. Anterior uveitis
B. Scleritis
C. Scleromalacia perforans
D. Peripheral corneal ulceration
E. Keratoconjunctivitis sicca

89) Which extraocular muscle inserts closest to the limbus?

A. Superior rectus
B. Medial rectus
C. Inferior rectus
D. Lateral rectus
E. Inferior oblique

90) A 69-year old man presents to your emergency department with an


acute retinal artery occlusion. His past medical history includes T2DM and
hyperlipidaemia only. Which intervention would be least appropriate?

A. Ocular massage
B. Hyperbaric oxygen
C. Intravitreal acetazolamide injection
D. Anterior chamber paracentesis
E. Thrombolysis
Paper 1 Answers

1) C
Cystoid macula oedema has many aetiologies which may be recalled via the
acronym DEPRIVEN: Diabetes, Epinephirine (C), Pars Planitis, Retinitis
pigmentosa, Irvine-Gass syndrome, Vein occlusion, E2 prostaglandin
analogues and Nicotinic acid.

2) D
All of the manifestations are associated with NF1 except myelinated nerve
fibres. Lisch nodules are melanocytic naevi of the iris and are typically
asymptomatic. There are a few key words worth remembering that might
point to a particular pathology. Optic nerve glioma - fusiform enlargement of
the optic nerve on CT, Rosenthal fibers on histology; plexiform neurofibroma
- ‘bag of worms’ on palpation, S-shaped ptosis of the upper lid.

3) C
The child has features of congenital syphilis (Hutchinson teeth and saddle
nose deformity) which causes interstitial keratitis. This type of keratitis
specifically affects the corneal stroma (C).

4) B
Important classic histology findings to be aware of are the rosette
appearance of retinoblastoma (A) and the characteristic peripheral palisading
of basal cell carcinoma (C). Other characteristic descriptions include keratin
“pearls” in well differentiated squamous cell carcinoma (B), shoulder
formation in keratoacanthoma (D) and vascular spaces with thin fibrous
septa in port wine stain (E).
5) B
The history of immunosuppression with a bilateral punctate epithelial keratitis
with spores on biopsy is characteristic of microsporidial keratitis (B).
Microsporidium is an obligate intracellular parasite which tends to affect
immunosuppressed individuals such as HIV positive people.

6) D
An important point to be aware of is this difference between a medical and a
surgical 3rd nerve palsy. Medical 3rd nerve palsy classically spares the pupil
whereas surgical 3rd nerve palsy affects the pupil first. This is because the
parasympathetic fibres of the oculomotor nerve run on the outside of the
nerve hence are more susceptible to compressive lesions such as aneurysms
(A) and herniations (B). The motor fibres run inside the 3rd nerve and are
supplied by the vasa nervorum; small blood vessels which are susceptible to
systemic disease affecting the vasculature. The commonest causes of which
are diabetes and hypertension (D).

7) D
Multiple capillary haemangiomas of the retina are virtually pathognomonic for
Von Hippel Lindau disease. VHL is a systemic AD inherited disease that is
characterized by cerebellar and renal tumours, polycythaemia and
endolymphatic sac tumours of the inner ear (D).

8) B
The lacrimal gland is supplied by the greater petrosal nerve which is a branch
of the 7th nerve which synapses at the pterygopalatine ganglion (B).
9) C
Trachoma is the leading cause of preventable irreversible blindness in the
world. Trachoma is caused by Chlamydia trachomatis serotypes A-C. Serovars
D-K cause conjunctivitis, urethritis and pelvic inflammatory disease.
Chlamydia is an obligate intracellular parasite but is also gram negative with
a peptidoglycan cell wall and causes trachoma via type IV hypersensitivity
reactions. Trachoma causes scarring of the palpebral conjunctiva which
causes the lid to rotate in towards the globe which is cicatricial entropion.

10) A
Pseudomonas is a troublesome infection that is more common in contact lens
wearers causing keratitis. Generally, antibiotics that are useful against
pseudomonas include aminoglycosides such as tobramycin (A), most
flouroquinolones except moxifloxacin and certain penicillins such as
carbapenems except ertapenem.

11) B
Down’s syndrome can give rise to several ophthalmic manifestations
including epicanthic folds, keratoconus, cataract, strabismus and spots of
connective tissue in the corneal stroma known as Brushfield spots (B).

12) A
The leading cause of blindness is cataract (A) affecting approximately 20
million people worldwide. However, if the question was asking the leading
cause of visual impairment worldwide; the most prevalent condition causing
visual impairment is refractive errors (D) which affects approximately 153
million people worldwide.

13) C
In Wilson’s disease copper is deposited in Descemet’s membrane (C) to give
rise to Keyser Fleischer rings.
14) C
The ‘sunset glow’ fundus is characteristic of Vogt-Koyanagi-Harada syndrome
(VKH). VKH is an autoimmune disease affecting melanocyte-containing
tissues, including choroid. Answers A, B & D are systemic associations with
retinitis pigmentosa. Gardner syndrome is associated with atypical congenital
hypertrophy of the RPE.

15) B
The history of blurring of central vision 6 weeks post cataract surgery is
suggestive of pseudo-phakic cystoid macular oedema AKA Irvine-Gass
syndrome (B). It is a common cause of visual impairment post-cataract
surgery and diabetics are at increased risk. Visual loss is usually transient and
peaks between 6-8 weeks and may be treated with topic NSAIDs or
corticosteroids.

16) E
The port-wine stain in the distribution of the trigeminal nerve branches (in
this case over the distribution of the ophthalmic division on the right side)
with a history of seizures is suggestive of Sturge-Weber syndrome. This is a
congenital sporadic phacomatosis with a number of ocular features including
choroidal haemangiomas, episcleral haemangiomas and open angle post-
trabecular glaucoma (E) which is caused by increased episcleral venous
pressure.

17) C
A person with a squint present in all directions and on primary gaze is likely
to have a concomitant manifest squint. The fact that the affected eye
adducts to take up fixation indicates it is outdeviated i.e. an extropia (C).
18) B
A sudden onset history of eye pain has a variety of differentials however the
presence of mutton fat keratitic precipitates points to sarcoidosis as the
cause (B) of an acute anterior uveitis.

19) A
The history of painful red eye with a crescent shaped ulcer at the corneal
limbus is very suggestive of peripheral ulcerative keratitis. It is associated
with autoimmune diseases of which, the history of joint pain makes
rheumatoid arthritis the likely cause (A).

20) D
The history of visual loss predominated by glares is characteristic of posterior
subcapsular cataracts (D). The glares are typically worse on miosis and
hence are worse when focusing on near objects. The longstanding history of
rheumatoid arthritis also means a long term steroid exposure is likely which
causes posterior subcapsular cataracts.

21) E
Aqueous humor is secreted by the ciliary epithelium (E). It is composed of
water, electrolytes and amino acids which helps it to nourish structures in the
anterior and posterior chambers.
22) E
The raised yellowish papular lesions located on the neck and flexural folds
classically occur in pseudoxanthoma elasticum with cutaneous laxity. The
ophthalmological sign that pseudoxanthoma is characterized by is angioid
streaks which are crack-like dehiscences in Bruch’s membrane appearing as
dark red linear lesions around the optic disc radiating from the peripapillary
area (E). Round oval lesions with yellowish subfoveal deposits (A) are egg-
like in appearance and hence are known as vitelliform macular degeneration
(AKA Best disease). Oval detachment of sensory macula (B) is a feature of
Central Serous retinopathy. Tigroid appearance of the retina (C) occurs in
degenerative myopia. The striae described in (D) are choroidal folds. Angioid
streaks are important in Duke Elder, other causes include Paget’s disease,
Sickle cell anaemia and Ehlers Danlos. Peau d’orange appearance with
angioid streaks is specific for pseudoxanthoma.

23) A
Oil droplet reflex and scissoring are the features of keratoconus on
examination with ophthalmoscopy and retinoscopy respectively. Another sign
that characterizes keratoconus are Fleischer rings (A). Fleischer rings (not to
be confused with Keyser-Fleischer rings of Wilson’s disease) are epithelial
iron deposits that surround the base of the cone. Mutton fat keratitic
precipitates (B) are a sign of sarcoid uveitis. Bull’s eye maculopathy (C) is the
characteristic appearance of chloroquine retinopathy. Angioid streaks (D) are
crack like dehiscences in Bruch’s membrane which are caused by:
pseuodoxanthoma elasticum, Ehler’s Danlos syndrome, Paget disease and
Sickle cell anaemia. Sunset glow fundus (E) is the appearance of the fundus
seen in Vogt-Koyanagi-Harada disease.
24) C
Cobblestone papillae are pathognomic of vernal conjunctivitis (C). Vernal
conjunctivitis is an IgE mediated disorder which is linked to atopy and
asthma. It tends to affect young boys in dry warm climates and may develop
to atopic keratoconjunctivitis.

25) B
The 3rd nerve originates at the level of the superior colliculus (A). It passes
between the superior cerebellar and posterior cerebral arteries (B) where it is
susceptible to compression from aneurysms. It pierces the pia mater anterior
and lateral to the posterior clinoid process (C) passing between the borders
of the tentorium cerebrii where it is susceptible to compression by
transtentorial uncal herniations. It traverses the lateral wall of the
cavernous sinus (D) superior to the other structures and divides into superior
and inferior segments before entering the superior orbital fissure (E).

26) A
Sickle cell anaemia which can cause either a proliferative or non-proliferative
retinopathy has several characteristic features. These features include:
angioid streaks, retina salmon patches which may later develop into black
sunburst retina lesions (A).
27) C
Supero-temporal ectopia lentis is classically seen in Marfan (C) disease, a
condition in which a genetic mutation affecting the gene responsible for
making fibrillin occurs causing connective tissue abnormalities. Fibrillin is an
important component of the Zonule of Zinn which suspend the lens to the
ciliary body hence lens dislocation occur when fibrillin is abnormal. Moreover,
Marfan disease can cause important cardiovascular lesions such as aortic root
dilatation, mitral valve prolapse and aortic aneurysm.

28) A
The best imaging modality for visualizing the extra-ocular muscles in thyroid
eye disease is a STIR (short tau inversion recovery) sequence (A). STIR
sequence is particularly useful for identification of active inflammation in
orbital tissues and often may show fusiform enlargement of extra-ocular
muscle bellies with tendon sparing in TED.

29) A
Pituitary lesions tend to affect the optic chiasm which contains decussating
fibers which supply the nasal retina on both sides hence causing bitemporal
hemianopia (A).

30) A
Tamsulosin is an alpha-1 adrenergic receptor blocker used to treat BPH. As
the iris has alpha-1 adrenoceptors it is also affected by the adrenergic
blockade effects of Tamsulosin. This causes the iris to become floppy intra-
operatively during cataract extraction, a condition known as Intraoperative
Floppy Iris Syndrome.
31) C
The inferior rectus tends to be most commonly affected by thyroid eye
disease. The muscles affected in order of how commonly they are affected by
thyroid eye disease: inferior rectus, medial rectus, superior rectus, levator
and lateral rectus (C). Lateral rectus is the least likely to be affected. The
acronym “I’M Slow” can be helpful to recall this order.

32) B
From among the many causes of avoidable blindness, five conditions have
been identified as immediate priorities within the framework of VISION
2020. The choice of these conditions is based on the burden of blindness
they represent and the feasibility and affordability of interventions to
prevent and treat them. These
are: cataract, trachoma, onchocerciasis, childhood
blindness and refractive errors and low vision.
Other disorders, such as glaucoma (B) and diabetic retinopathy, do not at
present meet all these criteria. However, as VISION 2020 progresses,
priorities are likely to include these challenges.

33) B
Redness of the eyes is known as hyperaemia or conjunctival hyperaemia. Of
the glaucoma medications listed the class that very commonly causes
conjunctival hyperaemia is prostaglandin analogues (B). Prostaglandin
analogues are the most efficacious drops in glaucoma however very common
side effect of conjunctival hyperaemia makes beta-blockers preferable.
Prostaglandin analogues may also cause iris hyperpigmentation and cystoid
macular oedema.
34) D
Keratoconus is the most common non-inflammatory corneal thinning disorder.
It has several systemic associations including: Turner syndrome, Marfan
syndrome, atopy, Ehler’s Danlos, Down syndrome and Leber’s congenital
amaurosis (D). Leber’s congenital amaurosis is a cause of perinatal blindness
in which an infant may exhibit an oculodigital reflex which entails poking
their eye. Persistent trauma to the corneal epithelium may lead to
keratoconus.

35) B
Systemic carbonic anhydrase inhibitors have several potentially serious side
effects hence tend to be reserved for acute treatments. Of all the listed side
effects, acetazolamide is least likely to cause hyperkalaemia (B) as it usually
causes the opposite (hypokalaemia).

36) E
Fuch’s uveitis syndrome (E) causes an inflammatory glaucoma. Fuch’s uveitis
is a chronic non-granulomatous uveitis which is characterized by iris
heterochromia and stellate keratitic precipitates.

37) D
The nasolacrimal duct is an important structure in the drainage of tear film
from the eye. Obstruction of this structure results in persistent eye watering
(epiphora). The length of the nasolacrimal duct is 12mm (D) and is opens
lateral and inferior to the inferior turbinate bone of the nose.
38) E
Microfilariae in onchocerciasis while alive largely do not cause pathology (E).
It is when the microfilariae die that they cause an inflammatory response by
releasing their antigens. Moreover onchocerciasis depends on a symbiotic
Wolbachia rickettsial bacteria to reproduce and a large proportion of the
immunogenic response to onchocerca death is thought to be due to this
bacteria. The other options listed are all correct.

39) E
LogMar visual acuity is a topic worth being familiar with. LogMAR stands for
Logarithm of Minimal Angle of Resolution. Because it is logarithmic an acuity
of 6/6 will correspond to a LogMAR of 0 whereas an acuity of 6/60 will
correspond to a LogMAR of 1. The WHO definition for blindness is acuity
worse than 3/60 which corresponds to a LogMAR of 1.3 or worse (where a
greater number on the LogMAR corresponds to worse vision) hence the
answer is (E).

40) A
The patient has a right sided internuclear ophthalmoplegia based on the
clinical findings (INO is on the side that fails to adduct on contralateral gaze
i.e. the right eye fails to adduct when looking to the left). Internuclear
ophthalmoplegia is associated with multiple sclerosis and although it is
unilateral in this case it can often be bilateral in the context of MS. A right
sided internuclear ophthalmoplegia is caused by a lesion to the right medial
longitudinal fasciculus (A).
41) E
Band keratopathy consists of deposition of calcium salts (E) in the corneal
Bowman layer, anterior stroma and basement epithelial membrane. It may
therefore occur in hypercalcaemia and is also seen in anterior uveitis.

42) D
Many aetiologies of bacterial keratitis are unable to penetrate the intact
corneal epithelium and as such most of the time bacterial keratitis will only
develop when ocular defences have been compromised. There are certain
organisms that can penetrate the corneal epithelium include: Neisseria
meningitidis, Neisseria gonorrhoeae, haemophilus and Corynebacterium
diphtheriae (D).

43) B
A Vossius ring is a deposition of pigment on the anterior lens capsule surface
(B) caused by significant blunt trauma to the eye.

44) D
The features of Horner’s syndrome are: partial ptosis, miosis, anhidrosis and
pseudo-enophthalmos. Partial ptosis is caused by loss of sympathetic tone to
the superior tarsal muscle and miosis due to loss of sympathetic input to the
iris dilator muscle. Enophthalmos in Horner’s is referred to as “apparent
enophthalmos” because the eye is not actually sunken back into the orbit but
the inactivation of the orbitalis muscle gives this appearance.

45) E
A Marcus-Gunn pupil is a term for the relative afferent pupillary defect sign
found on the swinging flashlight test of the pupils. It is caused by damage to
the optic nerve (E).
46) E
L-cones (red), M-cones (green) and S-cones (blue) are responsible for
trichromatic colour vision. Absence of one class of cones results in
dichromatic vision - protanopia (lack of red), deuteranopia (lack of green) or
tritanopia (lack of blue). When all three cone types are present but one type
is malfunctioning, this results in anomalous trichromacy - protanomaly (red
malfunction), deuteranomaly (green malfunction) or tritanomaly (blue
malfunction). The most common type of colour blindness is deuteranomaly.

47) B
There are a few genes worth remembering. TGFB1 gene is linked to a
number of corneal dystrophies. Myocilin (MYOC) and optineurin (OPTN)
mutations play a role in glaucoma. Other important genes include ABCA4
(Startgardt disease), complement factor H (CFH, age-related macular
degeneration) and RHO, RPGR & RP2 (retinitis pigmentosa). Mutations in
RPE65 cause Leber congenital amaurosis (autosomal recessive retinitis
pigmentosa) for which there are ongoing gene therapy trials with promising
results.
48) E
The nanny did a good job by immediately starting to irrigate the eye.
Severity of chemical eye injuries is graded by the Roper-Hall system. Pain
and subconjunctival haemorrhages are not included in the grading system.
The two signs that determine the grade are corneal transparency and limbal
ischaemia. Most severe (grade IV) injuries have a completely opaque cornea
or more than half of (i.e. >6 clock hours) of limbal ischaemia. Acid burns
(e.g. bleach) do not penetrate the tissue as deep as alkalis.

49) C
Optic nerve hypoplasia is characterized by the double ring sign on
fundoscopy which is a ring of white sclera surrounding a pigmented band
(A). Optic disc coloboma is a congenital defect of the optic disc that appears
as a discrete, focal, glistening white bowl shaped excavation of the optic disc
(C). Myelinated nerve fibers appear on fundoscopy as white striated patches
with feathery borders (D) and tilted optic discs appear as a small D shaped
disc with an axis directed infero-nasally (E). Funnel shaped excavation of the
optic disc is classic for the Morning glory anomaly (B).
50) A
Elschnig’s pearls appear after cataract surgery. They are clusters of residual
lens epithelial cells that migrate along the posterior capsule. All the other
answers are signs caused by iron deposition in tissues. Hudson-Stahli’s line is
found at the inferior 1/3 of the cornea and occurs due to aging. Fleisher ring
is a corneal epithelial iron ring around keratoconus. Stocker’s line is found at
the head of pterygium. Ferry’s line can be seen at the edge of filtering bleb
post trabeculectomy.

51) D
Given the examination findings, the most likely organisms are Aspergillus and
Fusarium causing filamentous keratitis. Fungi are best cultivated on
Sabouraud agar. In ophthalmological scenarios, McConkey agar is typically
used for Pseudomonas, chocolate agar for Gonococcus, E.coli seeded agar
plate for Acanthomoeba, and McCoy media for Chlamydia.
52) C
The key words to the diagnosis is Propionibacterium acnes and a white
capsular plaque. Interestingly, delayed-onset post-op endophthalmitis is
typically painless. If occurs >6 weeks post-op and is caused by bacteria
trapped within the capsular bag. Posterior capsular opacification is a
reasonable answer as it is the most common late complication after cataract
surgery, however it is caused by the proliferation and migration of residual
lens epithelial cells rather than bacteria. Phacoemulsification is a step in
cataract surgery when the lens is broken down using ultrasound and
aspirated - it is not a diagnosis! Anterior lenticonus is abnormal lens shape
associated with Alport syndrome. Sympathetic ophthalmitis would cause
symptoms in the sympathizing eye - the right eye.

53) E
Pachymetry is used to measure the CCT. The relevance of this in regard to
IOP measures is that thick corneas can give falsely high IOP readings,
whereas thin corneas might produce falsely low IOP readings. Perimetry
measures visual fields, specular microscopy looks at corneal endothelium,
biometry is used for intraocular lens
power calculation prior to cataract surgery.
54) B
Leber hereditary optic neuropathy has mitochondrial inheritance pattern.
Other hereditary fundus dystrophies worth knowing about are Startgardt
disease (autosomal recessive), Best disease (autosomal dominant),
choroideraemia (X-linked recessive), Stickler syndrome (autosomal
dominant).

55) C
The first histological change that happens in diabetic retinopathy is loss of
pericytes, followed by proliferation of endothelial cells and thickening of the
capillary basement membrane. The rest of the statements are correct. Cotton
wool spots form due to ischaemia and interruption of axoplasmic flow
resulting in cellular material accumulating within the nerve fibre layer. The
shape and direction of flame-shaped haemorrhages is due to the direction of
passage of the nerve fibres.
56) A
As concluded by the AREDS and AREDS2 studies, the current formula
recommends taking vit C, vit E, Zn, Cu, lutein and zeaxanthin in order to
decrease the risk of AMD progression.

57) C
Sclera is mainly made of collagen type 1 and type 3. The random
arrangement and interweaving of these fibres give the strength and flexibility
of the eyeball. Corneal stroma is also made primarily of collagen type 1 and
type 3, however the regular arrangement of these fibres account for
transparency of the tissue. Interestingly, in Bowman’s layer collagen types 1,
3, 5 and 6 are randomly arranged. The Descemet’s membrane is primarily
made up of collagen type 4. The vitreous contains collagen type 2, as well as
other structural proteins (hyaluronic acid, ascorbate, glycosaminoglycans).
58) C
Generally, which visual acuity test to chose for a paediatric patient depends
on the child’s age. <1 year old  Keeler cards (preferential looking); <1 
Cardiff acuity cards; 2-4  Kay pictures; >3  Sheridan-Gardner. Of
course, the choice of a test also depends on child’s maturity, co-operation,
learning ability etc. Hirschberg test is used as an initial screen for strabismus.
LogMAR is used in older children and adults.
59) E
Tyrosinase is an enzyme located in melanocytes and involved in melanin
synthesis. There are 2 forms of oculocutaneous albinism - tyrosinase-
negative (complete) and tyrosinase-positive (incomplete). Incomplete forms
retain ability to synthesize variable amounts of melanin and therefore are less
severe. Oculocutaneous albinism usually has autosomal recessive inheritance
pattern and hence can affect both genders. In contrast, ocular albinism is an
X-linked genetic disease, and is rarer than oculocutaneous forms. The rest is
true.

60) D
Yoke muscles are a pair of muscles (one muscle in each eye) moving the
eyes into the same direction of gaze, for example right medial rectus and left
lateral rectus work together to allow a left lateral gaze. Muscles in answer D
produce a gaze to the right & up.
61) B
The key to and answer in this question is description of a ‘clear zone’
strongly indicative of marginal keratitis or Terrien marginal degeneration.
However, in the latter the epithelium is intact, it is also rare and
asymptomatic. Marginal keratitis develops due to hypersensitivity reaction to
Staph exotoxins and is associated with chronic blepharitis. Other key words
worth being familiar with include ‘Wessely ring’ (disciform keratitis), ‘butterfly
pattern’ on topography (pellucid marginal degeneration). Answer E occurs in
patients with rheumatoid arthritis, granulomatosis with polyangiitis or SLE.

62) B
Japanese population has a substantially higher incidence of normal pressure
glaucoma compared to other ethnicities. Black race is a risk factor for primary
open angle glaucoma, whereas people from the far East (esp. Chinese) are
predisposed to primary angle-closure glaucoma. Pseudoexfoliation syndrome
(which can lead to glaucoma) is very common in Scandinavia.

63) A
Indirect lenses are high-plus lenses that are used in conjunction with a slit
lamp to produce a virtual image of the fundus that is laterally reversed and
inverted, producing an upside down, mirror image. One could argue that
answer E is the correct one, as a single cotton wool spot does not carry
much significance in HIV retinopathy - these are the commonest lesions and
tend to resolve spontaneously.
64) A
The vague history of gradual visual disturbance is non-specific. The
phenomenon of “second sight of the elderly” in which an elderly person with
presbyopia no longer requires their reading glasses is a classic history for a
nuclear sclerotic cataract (A). This is because the nuclear sclerotic cataract
consists of a central opacity which causes a myopic shift.

65) C
From the given drugs, atropine has the longest duration of action, followed
by homatropine, cyclopentolate and tropicamide. Pilocarpine is a myotic
agent and acts mainly on cholinergic M3 receptors found on the iris sphincter
muscle.

66) B
DVLA requires drives to have a visual acuity of 6/12 (aided or unaided) or
better in at least one eye. Drivers must also have satisfactory visual fields -
at least 120 degrees horizontally and extending at least 50 degrees left and
right, and there should be no significant defect in the binocular field that
encroaches within 20 degrees of the fixation (hence answer B is correct).
People with only one eye or colour-blind people are allowed to drive if they
meet the other minimum eyesight standards. Blepharospasm and diplopia
would also prevent people from driving.

67) B
3 steps to conversion: 1) add the cylinder power to the sphere 2) change the
cylinder positive/negative sign 3) change the axis by 90 degrees.
68) D
This is a difficult question requiring some basic understanding of
phototransduction. a wave - initial corneal negative deflection, derived from
the cones and rods of the outer photoreceptor layers. It reflects the
hyperpolarization of the photoreceptors due to closure of the sodium ion
channels in the outer-segment membranes (as light hyperpolarizes
photoreceptors). b wave - corneal positive deflection, derived from the inner
retina, predominantly Müller and ON-bipolar cells (as these depolarize when
rods and cones hyperpolarize). c wave - derived from the RPE and
photoreceptors.

69) A
About 95% of Birdshot’s patients are positive for HLA-A29. HLA-B27 is linked
to ankylosing spondylitis (associated with acute anterior uveitis) and Reiter’s
syndrome (associated with conjunctivitis and acute anterior uveitis). HLA-B51
is strongly linked to Behçet’s (associated with posterior uveitis).

70) D
Choroidal folds are typically associated with chronic papilloedema, orbital
disease, intraocular disease such as choroidal tumours and hypotony.
Congenital choroidal folds may be present in healthy, often hypermetropic
individuals. Other answers are clinical signs of degenerative myopia.
71) B
The ophthalmic artery has many branches that supply orbital and ocular
tissues, including the central retinal artery and ciliary arteries (long, short
and anterior). The cilioretinal artery is a branch of the short posterior ciliary
arteries and is considered to be a congenital anomaly. It supplies a portion of
the papillomacular bundle, even when the central retinal artery gets blocked.

72) C
Choroidal naevi are present in 5-10% of the Caucasian population. They are
typically found in the posterior equatorial and have indistinct margins (in
contrast to congenital hypertrophy of the RPE where lesions have well-
defined margins) Surface drusen may be present. Other answers are features
of potential malignancy (choroidal melanoma).

73) E
Drugs that can cause optic neuropathy include ethambutol, isoniazid,
amiodarone, chloramphenicol, hydroxychloroquine, vigabatrin, cisplatin,
vincristine, penicillamine.
74) B
The aqueous is secreted by the non-pigmented epithelium of the ciliary
processes at a rate of about 0.1 ml/hr. Most books quote that about 90% of
aqueous is drained by the trabecular route (which is pressure-sensitive) and
only about 10% by the uveoscleral route. Aqueous humour is produced via
active secretion using a Na/K ATPase dependent pump (2/3) and passive
ultrafiltration through blood vessels on the anterior surface of the iris (1/3).

75) D
The BRB is composed of both an outer and an inner barrier. The outer barrier
is formed at the RPE where tight junctions regulate the movement of solutes
and nutrients. The inner barrier comprises the microvascular endothelium,
which is non-fenestrated and forms tight junctions.
76) C
For the exam, you should be aware of two laws - Sherrington’s law and
Hering’s law. Sherrington’s law states that when one set of muscles is
stimulated, muscles opposing the action of the first are simultaneously
inhibited. Hering’s law states that during any conjugate eye movement, equal
and simultaneous innervation flows to the yoke muscles.

77) B
B correctly describes distichiasis. A refers to trichiasis, C - eyelash ptosis, D -
trichomegaly, E - madarosis.

78) A
Eales disease is an idiopathic peripheral retinal vasculopathy and is
hallmarked by recurrent vitreous hemorrhages. It would not cause
leukocoria, unlike other options.
79) D
The description is quite classical for blepharochalasis, even though it is a rare
disease. Floppy eyelids commonly affect obese middle-aged men who
complain of ocular irritation on awakening (as during the night the eyelid
everts and the inner surface rubs against the pillow). Epicanthus is a vertical
fold of skin on either side of the nose, sometimes covering the inner canthus.
Epiblepharon is an excessive lower lid tissue rotating the lower lid inwards.

Higher risk Lower risk


Superior break Inferior break
Retinal tear Retinal hole
Large break Small break
Symptomatic Detected by
Acute chance
tear/detachment Chronic
tear/detachment
80) A
The most important consideration in an acute RRD is whether the central
macula is involved. In ‘macula on’ detachments the central vision is
preserved and the surgery needs to be performed urgently in order to save
the central vision. In ‘macula off’ cases, where the macula is already
detached, the central vision can no longer be preserved and hence the
surgery is less urgent. Other considerations are shown in the table. Signs of
chronicity include pigmentation around the break, retinal thinning, intraretinal
cysts and subretinal demarcation lines (‘watermarks’). ‘Tobacco dust’ consists
of pigment cells in the anterior vitreous and is a clinical sign of a retinal
break.

81) E
The test described in the question is a Watzke-Allen test and the findings are
consistent with a macular hole. OCT is used for definitive diagnosis and
staging. Occasionally, macular holes close spontaneously. However, the most
effective treatment is vitrectomy.
82) B
A useful mnemonic for this question is ‘RPE Get Her (a) C A B Right Man’.
RPE (forms 1st), ganglion cells, horizontal cells, cones, amacrine cells, bipolar
cells, rods, Müller cells (forms last).

83) C
Retinal detachment describes a separation of the neurosensory retina from
the RPE. Answer B describes the RPE detachment and answer D describes
the retinoschisis.
84) C
Ora serrata is the junction between the retina and the ciliary body. Pars plana
is the posterior part of the ciliary body extending from the pars plicata to the
ora serrata. Schwalbe’s line marks the outer limit of the corneal endothelium
and represents the termination of Descemet’s membrane. Lamina cribrosa is
a mesh-like structure though which the optic nerve fibres leave the eye. The
grey line corresponds to the orbicularis oculi muscle in the eyelid margin.
85) E
ENDODERM MESODERM NEURAL ECTODERM
CREST NEURAL SURFACE
- vitreous - vitreous - NSR - lens
- choroid - choroid - ON - corneal
- not in eye - CB - CB - RPE epith
- TM + - TM + -
Schlemm’s Schlemm’s - extensions conjunctiva
canal - EOMs of NSR and - eyelids +
- EOMs - stromas RPE: lashes
- iris (e.g. iris,  CB
- ON corneal, epithelium - tear system:
sheath scleral)  iris lacrimal
- - corneal epithelium gland
orbicularis endothelium
oculi - iris nasolacrimal
- ocular - muscles syst
vascular Descemet’s meibomian
endoth membrane gland
- bony orbit
Learning embryology can be tricky, especially when different sources give
slightly different embryological origins of ocular structures. Extracellular
mesenchyme consists of both mesoderm and neural crest cells, and
structures formed from mesoderm are often infiltrated by neural crest cells
(e.g. as is the case with choroid). Therefore, for the DE exam we would
recommend learning the embryological origins of structures that are in bold,
which have less ambiguous and therefore ‘testable’ origins.
* CB - ciliary body; TM - trabecular meshwork; ON - optic nerve; EOMs -
extraocular muscles; NSR - neurosensory retina; RPE - retinal pigment
epithelium.
86) D
Evisceration is a surgical procedure when the cornea and the contents of the
globe are removed, leaving the sclera and extraocular muscles intact. It is
not suitable for suspected malignancy. Removal of the globe alone is called
enucleation. Removal of the globe and the soft tissue of the orbit is called
exenteration.

87) B
B is correct: Surgery + Antibiotics + Facial hygiene + Environmental
improvement.

88) A
Anterior uveitis is typically associated with seronegative (i.e. Rheumatoid
factor negative) arthritides. From the remaining options, keratoconjunctivitis
sicca is the most common ocular condition in rheumatoid arthritis patients.
89) B
Medial rectus inserts 5.5mm medial to the limbus. It is worth remembering
the spiral of Tillaux that describes the insertion of the rectus muscles in
relation to the limbus.

90) C
Acetazolamide is sometimes used in the acute retinal artery occlusion,
however this drug is given intravenously and cannot be given as an
intraocular injection. In acute retinal artery occlusion management options
include an attempt to dislodge the embolus (e.g. by lowering the IOP or
inducing vasodilation) or dissolve it (e.g. thrombolysis). Both acetazolamide
and paracentesis would lower the IOP. Ocular massage and hyperbaric
oxygen are also sometimes used.
Paper 2

90 MCQs
2 hours
1 ) Human eyes start to form at around 3 weeks post-fertilization. Which of
the following options is the correct sequence of eye development?

A. Optic vesicle  Optic groove & lens placode  Optic cup


B. Optic groove  Optic vesicle & lens placode  Optic cup
C. Optic stalk  Optic vesicle & lens placode  Optic cup
D. Choroidal fissure  Optic groove & lens placode  Optic cup

2) Which of the following germ layers contributes to the development of


ocular structure the least?

A. Surface ectoderm
B. Neural crest
C. Mesoderm
D. Endoderm

3) Which of the following clinical signs would be in keeping with corneal graft
rejection?

A. Hassall-Henle bodies and Stocker’s line


B. Hudson-Stähli line
C. Krachmer spots and Khodadoust line
D. Fleisher ring
4) You are reviewing a patient who has undergone penetrating keratoplasty 8
months ago and is now complaining of decreased vision and increasing
discomfort. On examination of the cornea you can see the sutures and the
overlying epithelium appears intact. However, you notice some grey-white
branching stromal opacities with minimal inflammation. Which of the
following is most likely to be responsible for these findings?

A. Filamentary keratitis
B. Bacterial infection
C. Normal healing process
D. Poor adherence to steroid eyedrops

5) Which of the following is the commonest pathogen causing keratitis in


contact lens wearers?

A. Pseudomonas aeruginosa
B. Staphylococcus aureus
C. Aspergillus fumigatus
D. Acanthamoeba

6) Fundus fluorescein angiography (FFA) is a widely used imaging modality to


evaluate retinal disease. From the option below, which retinal condition
would show a hypofluorescent defect on the FFA?

A. Cystoid macular oedema


B. Central serous retinopathy
C. Macular hole
D. Preretinal haemorrhage
7) You examine a 29-year-old man complaining of visual problems on a slit
lamp. On examination of the lens, you can see polychromatic needle shaped
opacities. Which medical condition is associated with these findings?

A. Type 1 Diabetes Mellitus


B. Myotonic dystrophy
C. Severe atopic dermatitis
D. Neurofibromatosis type 2

8) The retinal pigment epithelium (RPE) is vital to the normal function of the
neural retina. Which of the functions the RPE does NOT perform?

A. Aids generation of ON and OFF retinal signaling pathways


B. Production and recycling of photopigments
C. Phagocytosis of photoreceptor discs
D. The maintenance of the outer blood-retinal barrier

9) Which of the following statements is correct regarding diabetic


retinopathy?

A. Thickening of the capillary basement membrane is the first histological


change
B. Hard exudates are the earliest clinical sign seen on fundoscopy
C. Hard exudates accumulate within the inner plexiform layer (IPL) of the
retina
D. Cotton wool spots are focal accumulations of neuronal debris within the
nerve fiber layer
10) In your glaucoma clinic, the next patient is a 42-year-old Caucasian lady.
She has primary open-angle glaucoma (POAG) and has come for a follow-up.
You look through her notes before seeing her. From the list below, which of
the factors are known to increase her risk of POAG?

A. Her glass prescription is OD +4.00D and OS +4.25D


B. She is diabetic
C. She is Caucasian
D. Her central corneal thickness is 580 microns

11) Aqueous humour is produced by the ciliary body. From the posterior
chamber it moves though the pupil to enter the anterior chamber and exits
the eye via the trabecular and uveoscleral routes. Which statement about the
aqueous humour physiology is correct?

A. Aqueous humour is secreted by the pigmented epithelium of the ciliary


processes
B. Aqueous humour flow follows a circadian rhythm
C. Uveoscleral route is pressure sensitive
D. Carbonic anhydrase inhibitors increase the aqueous humour outflow
12) A 52-year-old woman has been recently diagnosed with a large pituitary
adenoma after presenting with increasing headaches and problems with her
vision. She has been referred to a neuro-ophthalmology clinic due to
bitemporal hemianopia and a new onset nystagmus. Which type of
nystagmus is she most likely to have?

A. Downbeat nystagmus
B. Upbeat nystagmus
C. See-saw nystagmus
D. Convergence-retraction nystagmus

13) Retinitis pigmentosa (RP) is a group of retinal dystrophies. Occasionally it


is associated with systemic diseases or syndromes. From the list below,
which of the following is NOT associated with RP?

A. Kearns-Sayre syndrome
B. Larsen syndrome
C. Refsum disease
D. Usher syndrome
14) You are a newly appointed ophthalmology trainee in a retinal clinic and
you are examining a patient with bull’s-eye maculopathy secondary to
chloroquine toxicity. Which one of the following conditions can also give rise
to bull’s-eye maculopathy?

A. Tay-Sachs disease
B. Sialidosis
C. Niemann-Pick disease
D. Stargardt disease

15) A 7-year-old boy is brought by his dad to see an ophthalmologist. He is


constantly rubbing his eyes because they are ‘itchy’. This has happened
before but has never been so troublesome. You can see from the distance
that both eyes are red and watery. Slit lamp examination reveals peri-limbal
small white dots, more prominent superiorly. Which other clinical finding
would be consistent with the most likely diagnosis?

A. Giant papillary conjunctivitis


B. Follicular conjunctivitis
C. Red-white conjunctival masses of wool-like consistency
D. Preauricular lymphadenopathy
16) You are an ophthalmology trainee who has been asked to review a pre-
term baby on the neonatal intensive care unit who developed signs of
bacterial conjunctivitis on day 2 postpartum. Which pathogen is the most
likely cause?

A. Staphylococcus aureus
B. Chlamydia trachomatis
C. Neisseria gonorrhoeae
D. Pseudomonas aeruginosa

17) Which of the following statements regarding retinopathy of prematurity


(ROP) is true?

A. In the UK, all premature newborns should be screened for ROP


B. ROP is most commonly treated with intravitreal anti-VEGF injections
C. Total retinal detachment is the final and most severe stage of ROP
D. ROP can show spontaneous regression if high concentration of
supplemental oxygen is used

18) In 1964, Duke Elder described a ‘symmetrical enlargement of the


palpebral aperture occurring as a primary abnormality associated with large
eyelids’. Which of the following was he referring to?

A. Euryblepharon
B. Symblepharon
C. Ankyloblepharon
D. Epiblepharon
19) In which part of the human brain is the horizontal gaze center found?

A. Rostral interstitial nucleus


B. Paramedian pontine reticular formation
C. Frontal eye fields
D. Superior colliculus

20) You review a 5-year-old girl in a follow-up clinic. Her visual acuity
remains normal. On examining her eye movements, you notice that when
she attempts to look to the right, the left eye fails to adduct fully and the left
palpebral fissure becomes narrower. Which of the following is the most likely
diagnosis?

A. Left internuclear ophthalmoplegia


B. Parinaud syndrome
C. Brown syndrome
D. Duane syndrome

21) There are a few ways to assess one’s visual acuity in adults, for example
Snellen’s chart, LogMAR chart, and duochrome test. Which of the following
statements regarding the duochrome test is correct?

A. It can be used in colour-blind patients


B. Red light is refracted more than green light as it passes through the lens
C. Is only sensitive to changes in refraction of 0.50D or more
D. It relies on principle of contrast sensitivity
22) You are in your early years of ophthalmology training and have been
really enjoying your life, but now you need to start preparing for your
refraction and optics exams. You are reading a chapter on direct and indirect
ophthalmoscopy. Which of the following statements describes properties of
indirect ophthalmoscopy, rather than direct ophthalmoscopy?

A. Indirect ophthalmoscopy has a higher magnification factor


B. Indirect ophthalmoscopy produces an inverted image
C. Indirect ophthalmoscopy has a smaller field of view
D. Indirect ophthalmoscopy produces a flat two dimensional image

23) You are learning how to assess the iridociliary angle. You are examining a
patient on a slit lamp using the goniolens. In the angle, you can see a few
lines next to each other. What is the correct anatomical order of the
structures seen through the goniolens?

A. Scleral spur  Trabecular meshwork  Schwalbe’s line  Ciliary body


B. Scleral spur  Schwalbe’s line  Trabecular meshwork  Ciliary body
C. Schwalbe’s line  Scleral spur  Trabecular meshwork  Ciliary body
D. Schwalbe’s line  Trabecular meshwork  Scleral spur  Ciliary body
24) A 9-year-old boy is seen in epilepsy clinic. Apart from occasional seizures,
he is also known to have learning difficulties. The boy has red spots on his
face in a butterfly distribution, and a couple of lumps emerging from the nail
folds of his toes. The neurologist decides to refer him to ophthalmology, even
though he does not have any visual symptoms. Which of the following
features are most likely to be seen during his fundus exam?

A. Clusters of saccular aneurysms resembling a bunch of grapes


B. Oval red lesions located between an arteriole and venule
C. Large yellowish elevated mulberry-like lesions
D. Yellowish flat well-defined lesion with scalloped borders

25) Today you’re working in uveitis clinic. Your consultant is reviewing a 23-
year-old woman, following her initial presentation to the eye casualty with a
red eye. He asks you to come and examine the anterior segment of the
patient, as ‘you don’t often get to see this classic sign’. On examination, you
can see some keratic precipitates, but the most striking feature is a few
clusters of dilated iris capillaries. Which bacteria is the most likely cause of
this presentation?

A. Bartonella henselae
B. Mycobacterium leprae
C. Treponema pallidum
D. Borrelia burgdorferi
26) Phototransduction is a complex process during which photoreceptors
convert light energy into an electrical signal. Which of the following
statements regarding phototransduction is true?

A. Light converts all-trans retinal to 11-cis retinal


B. Phosphodiesterase converts GMP to cGMP
C. Sodium channels are closed if cGMP is bound to them
D. Closure of sodium channels causes hyperpolarization

27) Which of the following retinal clinical signs are found in acute
hypertensive crisis?

A. Copper and silver wiring


B. Elschnig spots and Siegrist streaks
C. Choroidal folds
D. Angioid streaks

28) A 42-year-old banker is seen in the retinal clinic. He is complaining of


progressively worse blurred vision with slight distortions. He ‘was too busy at
work’ to seek eye assessment earlier. When examining his fundus, you note
that there is an oval detachment of the macula. You order a fluorescein
angiography that confirms your suspected diagnosis. What did fluorescein
angiography show?

A. Inkblot leakage pattern


B. Flower petal pattern
C. Lacy pattern with later leakage and staining
D. Hyperfluorescent crests and hypofluorescent troughs
29) Which bones contribute to the medial wall of the bony orbit?

A. Maxilla, sphenoid, lacrimal and nasal


B. Maxilla, sphenoid, ethmoid and lacrimal
C. Maxilla, ethmoid, lacrimal and frontal
D. Maxilla, sphenoid, ethmoid, lacrimal and nasal

30) Which of the following actions does the inferior oblique muscle perform?

A. Elevation, adduction and extorsion


B. Elevation, adduction and intorsion
C. Elevation, abduction and extorsion
D. Elevation, abduction and intorsion

31) A 35-year-old lady presents to the eye casualty department at 7.30 am


wearing sunglasses. She complains of severe burning and grittiness of her
left eye. On questioning, these symptoms have come and gone intermittently
over the past 2 years, but this episode is particularly bad. On examination
with the slit lamp, you notice a crust at the bases of all the eyelashes. What
is the most commonly associated condition?

A. Acne rosacea
B. Seborrhoeic dermatitis
C. Atopic dermatitis
D. Demodex follicularum brevis
32) A 1-day history of unilateral epiphora with pericanalicular redness and
oedema is most likely to be caused by which of the following micro-
organisms?

A. Staphylococcus aureus
B. Actinomyces israelii
C. Herpes simplex virus
D. Streptococcus pneumonia

33) A 66-year-old woman presents to A/E with vomiting and a right sided
headache. Her observations are stable on admission and she is noted to have
a red right eye with appears dilated on gross examination. Which of the
following is a risk factor for her likely condition?

A. Exposure to bright lights


B. Myopia
C. Female
D. Caucasian ethnicity

34) A 54-year-old woman complains of gradual impairment of her central


vision and flashes of light. Her fundoscopy reveals multiple cream coloured
ovoid lesions radiating from the optic disc. OCT reveals cystoid macular
oedema. Which of the following investigations would be most useful in this
patient?

A. Serum ACE
B. HLA – A29
C. Hb electrophoresis
D. MRI head
35) A 23-year-old man injures his neck whilst skateboarding and suffers from
neck pain and stiffness afterwards. He is seen in A/E afterwards and on
general examination he is noted to have anisocoria. In the dark his right
pupil is 5mm whereas his left pupil is 1.5mm. What is the location of the
lesion responsible for his difference in pupil size?

A. Tentorium cerebri
B. Posterior communicating artery
C. Ciliary body
D. Internal carotid artery

36) A 30-year-old man complains of gradual vague visual disturbance. He has


no previous ocular history of note and on slit lamp examination of his eye
you note a shield like anterior-subcapsular lens opacity. Which of the
following is most likely to establish his diagnosis?

A. Drug history
B. History of previous episodes of DKA
C. History of contact lens use
D. History of severe eczema

37) What is the commonest cause of unilateral proptosis?

A. Carotid-cavernous fistula
B. Orbital cellulitis
C. Retrobulbar haemorrhage
D. Grave’s disease
38) A 33-year-old man complains of gradual loss of vision. On fundoscopy
you note that he has a foveolar zone of pigment surrounded by a
depigmented RPE with an outer hyperpigmented layer resembling a bull’s
eye. Which of the following is most likely to have caused his visual
deterioration?

A. Phenytoin toxicity
B. Hydroxycholoroquine toxicity
C. Best’s disease
D. Vogt-Koyangi Harada disease

39) A 2-year-old Japanese boy is seen in clinic. His parents are concerned
that his eyes appear misaligned. He was born at 40 weeks gestation with no
complications. He has no past medical history. The Hirshberg reflex is
centred in the pupils. What is the most likely underlying condition?

A. Esotropia
B. Esophoria
C. Pseudoesotropia
D. Retinoblastoma

40) Which of the follow intra-ocular muscles is capable of both abduction and
elevation of the globe?

A. Superior rectus
B. Lateral rectus
C. Superior oblique
D. Inferior oblique
41) A 43-year-old woman suffering from ophthalmoplegia for several months
undergoes a Hess chart test. The Hess chart shows that her left visual field is
hypotropic on primary gaze with compression of the field reflected by
restriction to upgaze and abduction in the left eye. What is the most likely
diagnosis?

A. Left oculomotor palsy


B. Left abducens palsy
C. Left thyroid eye disease
D. Left Duane syndrome

42) A 39-year-old woman reports a 1-day history of binocular diplopia


associated with nausea and headache. She reveals that she had previously
experienced a bout of diarrhoeal illness. On further examination you note
that she demonstrates ataxia and areflexia of the lower limbs. Which of the
following is most likely show an abnormality?

A. USS renal tract


B. MRI head
C. Anti-GQ1b
D. Fundoscopy

43) A 13-year-old boy presents with gradual onset unilateral ptosis which has
progressed to bilateral ptosis. On examination he has some degree of
ophthalmoplegia in both eyes. Your consultant performs fundoscopy and
instructs you to obtain an ECG for the child. What did he see on inspection of
the fundus?

A. Salt and pepper retinopathy


B. Retinitis pigmentosa
C. Bull’s eye maculopathy
D. Macular star
44) A 60-year-old man complains of visual impairment of his right eye. An
OCT scan is performed shows retinal thickening with intraretinal cystic areas.
Which of the following is a risk factor for his condition?

A. Age
B. Raised intra-ocular pressure
C. Hypertension
D. Retinitis pigmentosa

45) A 20-year-old man with Down’s syndrome complains of blurred vision in


his left eye. On examination you note bulging centrally thinned corneas with
a scissoring reflex on retinoscopy. Given the likely diagnosis which of the
following signs is most consistent with the man’s condition?

A. Fleischer rings
B. Arlt’s line
C. Optic disc oedema
D. Band keratopathy

46) What is the axial length of the normal neonate human eye?

A. 16mm
B. 18mm
C. 20mm
D. 22mm
47) What is the commonest cause of visual impairment worldwide?

A. Cataract
B. Glaucoma
C. Trachoma
D. Refractive errors

48) A 56-year-old woman complains of headache and double vision. On


further questioning she reports her headache is right sided and located
around her eye. On fundoscopy you note the optic disc appears chalky white
and oedematous. What is the most appropriate initial treatment?

A. IV acetazolamide
B. Intravitreal triamcinolone
C. Oral prednisolone
D. Topical pilocarpine

49) A 10-day old neonate is brought to A/E with a red right eye discharging
pus. On inspection there is a hyperpurulent discharge from the right eye with
severe eyelid oedema. Given the likely diagnosis which organism is likely to
be implicated?

A. Staphylococcus aureus
B. Chlamydia trachomatis
C. Haemophilus influenzae
D. Neisseria gonorrhoea
50) A lens with a focal length of 25cm has a dioptric power of:

A. 25 dioptres
B. 2.5 dioptres
C. 4 dioptres
D. 0.4 dioptres

51) A 43-year-old Caucasian man is seen in clinic with a shiny firm pearly
nodule with small overlying blood vessels on his lower eyelid adjacent to the
medial canthus. Your consultant advises you to obtain a biopsy of the lesion.
What characteristic appearance would you expect to see on histopathology?

A. Rosettes
B. Keratin pearls
C. Shoulder formation
D. Peripheral palisading

52) On a routine check-up of a 2-year-old child the GP notes a round sharply


delineated macular lesion of the right eye on fundoscopy. He describes the
lesion as vitelliform. The child has no visual symptoms. What is the most
likely diagnosis?

A. Stargardt disease
B. Best disease
C. Alport syndrome
D. Leber’s congenital amaurosis
53) What is the correct definition of a prism dioptre?

A. 1 prism dioptre produces 1cm visible displacement at 100cm distance


towards the prism base
B. 1 prism dioptre produces 1cm visible displacement at 100cm distance
towards the prism apex
C. 1 prism dioptre produces 1cm visible displacement at 50cm distance
towards the prism base
D. 1 prism dioptre produces 1cm visible displacement at 50cm distance
towards the prism apex

54) A 63-year-old gentleman is seen in clinic with some gradual onset visual
impairment. On slip lamp examination you note he has bilateral cataracts. His
acuity is 6/18 in the right eye and 6/12 in the left. His acuity improves when
wearing glasses to 6/12 in the right eye and 6/6 in the left eye. He asks you
whether he can drive his car. What is the most appropriate response?

A. He must stop driving immediately


B. He is safe to drive only if he wears his glasses
C. He must stop driving and have his acuity rechecked after cataract surgery
D. There is insufficient information to give advice
55) A 67-year-old man is found incidentally to have an abnormal appearance
of his retina on fundoscopy. He has no visual impairment however has
reported that his hearing is worsening and that he has been suffering back
pain and reports no family history of disease. Dark red linear lesions with
irregular serrated edges that intercommunicate in a ring-like fashion around
the optic disc are seen. Which of the following investigations would be most
useful to confirm his likely diagnosis?

A. LFTs with calcium and phosphate


B. CT head
C. MRI head
D. OCT

56) A 40-year-old woman is found to have fine golden-brown opacities in her


cornea forming a whorled pattern. She reports no symptoms. She has a past
medical history of atrial fibrillation, Type II diabetes, previous TIA and
Neurofibromatosis Type 1. What is the most likely cause of the appearance of
her cornea?

A. Type II diabetes
B. Clopidogrel
C. Amiodarone
D. Insulin
57) A 65-year-old woman with a history of glaucoma treated with eye drops
reports her eyes becoming increasingly red. She is otherwise asymptomatic.
What is the most likely cause of her red eyes?

A. Topical timolol
B. Topical bromonidine
C. Topical latanoprost
D. Allergic reaction to medications

58) A neonate is found to have an abnormal appearance of their retina on


fundoscopy. On inspection you note a double ring at the optic disc with a
inner pigmented band and outer white ring. The baby was born at 40 weeks
by a normal delivery with no complications. The mother has a past history of
Type 1 diabetes. What is the most likely diagnosis?

A. Tilted optic disc


B. Morning glory abnormality
C. Myelinated nerve fibres
D. Optic nerve hypoplasia
59) A 24-year-old man is hit in the eye with a tennis ball. He has peri-orbital
ecchymoses and restricted movement of his eyeball in up and downgaze. He
has no impairment of vision. Given his likely diagnosis which of the following
structures is most commonly affected?

A. Orbital roof
B. Orbital floor
C. Medial wall of orbit
D. Lateral wall of orbit

60) Which of the following bones does not contribute to the medial wall of
the orbit?

A. Ethmoid bone
B. Lacrimal bone
C. Maxillary bone
D. Nasal bone

61) A 45-year-old man complains of blurring of his vision in the left eye. On
examination of his eyes you note that he has a posterior subcapsular cataract
in the left eye. You also note that his left iris appears hypochromic compared
to his right eye and there are small nodules on the inner surface of his left
iris. Which other sign would be most consistent with his underlying
diagnosis?

A. Miosis
B. Ptosis
C. Ciliary injection
D. Stellate keratitic precipitates
62) What is the thickness of a normal central cornea?

A. 510 µm
B. 530 µm
C. 550 µm
D. 570 µm

63) A 22-year-old lady presents to eye casualty with diplopia in the right eye
that is associated with severe pain worsened by eye movements. A CT scan
is done, which shows enlargement of the right medial rectus muscle and its
tendon. What is the most likely cause?

A. Orbital myositis
B. Thyroid eye disease
C. Myosarcoma
D. 3rd nerve palsy

64) On autofluorescence imaging, what colour is the optic disc?

A. White
B. Red
C. Black
D. Yellow

65) At what age does the eyeball stop growing?

A. At birth
B. 2 years old
C. 10 years old
D. 22 years old
66) What is the leading cause of preventable irreversible blindness
worldwide?

A. Cataract
B. Glaucoma
C. Trachoma
D. Age-related macular degeneration

67) Which of the following types of lens would be most useful for a myope?

A. Biconcave
B. Biconvex
C. Toric
D. Straight

68) A 30-year-old gentleman living in London has a 2-day history of blurred


vision and extreme pain in his right eye, and as a result has been unable to
tolerate his contact lenses. On slit lamp examination, you notice some
infiltrates along the corneal nerves. Some corneal scrapings are taken to
send off for culture. Which of the following cultures is most likely to grow the
causative organism?

A. McCoy agar
B. Sabouraud agar
C. Non-nutrient agar with E.coli
D. MacConkey agar
69) Measurement of which combined ocular parameters are most important
to calculate the intraocular lens power needed during cataract surgery?

A. Keratometry and tonometry


B. Axial length and keratometry
C. Refractive error and keratometry
D. Refractive error and keratometry

70) Which of the following is the most common cause of congenital cataract?

A. Chromosomal abnormalities
B. Autosomal dominant inheritance
C. Metabolic disorders
D. Intrauterine infections

71) A 20-year-old gentleman with coarse blonde hair, blue irides, learning
difficulty, and a marfanoid habitus presents with ectopia lentis. In which
direction is the lens most likely to be displaced?

A. Superotemporal
B. Inferonasal
C. Inferotemporal
D. Superonasal
72) Which of the following describes the correct extension of the visual field?

A. 50o superiorly, 60o nasally, 70o inferiorly, 90o temporally


B. 70o superiorly, 60o nasally, 80o inferiorly, 90o temporally
C. 50o superiorly, 50o nasally, 90o inferiorly, 70o temporally
D. 40o superiorly, 50o nasally, 65o inferiorly, 85o temporally

73) Which one of the following chemicals is MOST likely to be associated with
an acute elevation of IOP and longstanding ocular damage?

A. Chlorine bleach
B. Sulfuric acid
C. Sodium hydroxide
D. Hydrogen peroxide

74) A 67-year-old man has raised intra-ocular pressure of 35mmHg. The eye
is white and the anterior chamber is deep. You notice fluffy, dandruff-like
deposits on the anterior lens capsule. What is the diagnosis?

A. Self-resolving angle-closure glaucoma attack


B. Primary open angle glaucoma
C. Pseudoexfoliative glaucoma
D. Pigment dispersion glaucoma
75) A patient has uveitis. The ophthalmologist describes the appearance of
the retina through the slit lamp. There is peripapillary atrophy, multiple white
chorioretinal ‘spots’, and no evidence of vitritis. What is the cause?

A. Presumed ocular histoplasmosis syndrome


B. Seronegative arthropathy
C. Sarcoidosis
D. CMV retinitis

76) Which of the following is true regarding Vogt-Koyanagi-Harada


syndrome?

A. It is most common in Caucasian individuals


B. Dalen-Fuchs nodules lie between the retinal pigment epithelium and
sensory retina
C. The ‘sunset glow’ fundus appears during the acute phase
D. Neurological and auditory manifestations usually resolve

77) Which condition is related to capillary haemangioma?

A. Tuberous sclerosis
B. Von Hippel Lindau
C. Retinoblastoma
D. Cavernous haemangioma
78) Which of these correctly defines high myopia?

A. Refractive error of more than +5 dioptres


B. Refractive error of more than -3 dioptres
C. Refractive error of more than -6 dioptres
D. Refractive error of more than -8 dioptres

79) Which of these variances in the optic chiasm occurs in ocular albinism?

A. There are fewer crossed nerve fibres than normal


B. There are more crossed nerve fibres than normal
C. There are fewer nerve fibres than normal
D. There are more nerve fibres than normal

80) Which of these is true of the spiral of Tillaux?

A. The medial rectus inserts closest to and the lateral rectus insets furthest
away from the corneal limbus
B. The inferior rectus origin forms part of the spiral
C. The superior rectus inserts 7.7mm from the corneal limbus
D. The medial rectus inserts 4.5mm from the corneal limbus
81) Six prism dioptres of left esotropia are best corrected by:

A. 3 dioptres base-in left eye, 3 dioptres base-in right eye


B. 3 dioptres base-out left eye, 3 dioptres base-out right eye
C. 6 dioptres base-out left eye
D. 3 dioptres base-out left eye, 3 dioptres base-in right eye

82) For a healthy 15-month-old toddler, which is the most appropriate test of
visual acuity:

A. Cardiff acuity cards


B. Kay pictures
C. Sheridan-Gardner chart
D. Forced preferential looking

83) The image formed by a prism is:

A. Erect, real, displaced towards the base


B. Inverted, virtual, displaced towards the base
C. Erect, real, displaced towards the apex
D. Erect, virtual, displaced towards the apex
84) A patient presents with bilateral sixth nerve palsy. Where is the most
likely location of nerve compression?

A. Clivus
B. Cavernous sinus
C. Superior orbital fissue
D. Cerebello-pontine angle

85) A patient suffers from left exotropia. On the cover test, when the left eye
is covered:

A. The right eye will move outwards and subjectively the image on the right
will disappear
B. The right eye will stay stationary and subjectively the image on the left
will disappear
C. The right eye will stay stationary and subjectively the image on the left
will disappear
D. The right eye will move inward and subjectively the image on the right will
disappear

86) A child has a left eye that has been invaded by a retinoblastoma. Which
of the following procedures is most appropriate?

A. Enucleation
B. Evisceration
C. Exenteration
D. Radiotherapy
87) Which is required to legally drive in the United Kingdom?

A. Acceptable vision in both eyes


B. Good stereopsis
C. At least 120 degrees of horizontal visual field
D. Ability to read a number plate from 17.5 meters

88) Which is the most common primary malignant site to metastasise to the
orbit in females?

A. Bronchus
B. Breast
C. Brain
D. Skin

89) What is the most common cause of amblyopia?

A. Refractive errors
B. Strabismus
C. Congenital cataract
D. Trachoma
90) Which of the following nerves passes through the foramen ovale?

A. Ophthalmic division of the trigeminal nerve


B. Greater petrosal nerve
C. Otic ganglion
D. Ophthalmic veins
Paper 2 Answers

1) B
Eyes form from the diencephalon. At 3 weeks the optic grooves appear,
and at about 4 weeks they become the optic vesicles. When the optic
vesicles reach the outer surface of the neural tube, they induce lens
placode formation, which will give rise to the lens. Once the optic vesicle
reaches the outer surface, the lens placode starts folding into the optic
vesicle. The vesicle now becomes the optic cup. At the same time, the
hyaloid artery starts pushing into the optic cup from below. This C-
shaped cleft is called the choroidal fissure. The hyaloid artery supplies
the developing eye. It is worth knowing about a few pathologies that can
arise during developmental stages: pupillary membrane, coloboma,
Bergmeister’s papilla and Mittendorf’s dot.

2) D
It is crucial to learn the germ layer origins of various tissues. Refer to the
table on page 57. The endoderm is the germ layer that contributes least
to the ocular tissues.
3) C
Hassall-Henle bodies are small transparent growths posterior to
Descemet’s membrane at the periphery of the cornea and appear with
normal ageing. Stocker’s line is a pigmented epithelial iron line at the
head of the pterygium. Hudson-Stähli line is another iron line of the
corneal epithelium located at the junction of the upper 2/3 and lower 1/3
of the cornea, as a result of tear pooling. Fleisher ring is iron deposition
in corneal epithelium seen in keratoconus. Krachmer infiltrates and
Khodadoust are both seen in corneal graft rejection.

4) B
The vignette fits the diagnosis of infectious crystalline keratopathy, most
commonly caused by Streptococcus viridans. It is seen in patients who
have prolonged topical steroids, especially after penetrating keratoplasty
(which often required steroid drops to be administered for 1 year!).
Fungal keratitis, although not an option here, could be another possible
differential as it is also associated with long-term topical steroids.
However, the question would most likely mention stromal infiltrates with
‘fluffy margins, feathery extensions and satellite lesions’. Filamentary
keratitis is ocular surface disease.
5) A
Pseudomonas aeruginosa is the most common pathogen causing
bacterial keratitis in both patients with and without contact lenses.
Acanthamoeba is indeed most frequently associated with contact lens
wear. However, Acanthamoeba keratitis is rare and remains much less
common than Pseudomonas infection in contact lens users.

6) D
FFA is used to assess the choroid and retinal vasculature.
Hypofluorescent defects occur either due to a transmission defect, i.e.
blockage of normal fluorescence (e.g. corneal scar, preretinal
haemorrhage), or due to a filling defect (e.g. arterial occlusion).
Hyperfluorescent defects result from the following: window defect, where
RPE is absent and choroid shines through (e.g.macular hole); leakage of
dye (e.g. cystoid macular oedema); pooling of dye (e.g. central serous
retinopathy); staining of dye (e.g. drusen); abnormal vessels (e.g.
tumours); autofluorescence (e.g. drusen, lipofuscin).
7) B
This question focuses on cataract in systemic disease. The following lens
findings are seen in systemic conditions: DM - snowflake cortical
opacities; MD (B) - fine cortical iridescent opacities, polychromatic needle
opacities (Christmas tree), stellate posterior subcapsular cataract; NF2 -
posterior subcapsular cataract; Wilson’s disease - green-brown
pigmentation sunflower cataract; galactosaemia - central oil droplet
opacity; mannosidosis - spoke-like posterior cataract. Remember the
cataract resulting from a blunt trauma - flower-shaped opacity, rosette.

8) A
Other RPE functions include storage of metabolites and vitamin A,
nutrient supply to the photoreceptors and absorption of scattered light
by melanosomes.

9) D
Histological changes in diabetic retinopathy occur in the following order:
loss of pericytes  proliferation of endothelial cells  thickening of
the capillary basement membrane. The earliest clinical sign of diabetic
retinopathy is microaneurysms. Hard exudates form within the outer
plexiform layer (OPL) when these microaneurysms start leaking.
10) B
Risk factors for POAG are high IOP, increasing age, Black ethnicity, family
history (1st degree relatives), myopia and systemic vascular disease (e.g.
diabetes mellitus). Risk factors for primary angle-closure glaucoma
(PACG) include age, far Eastern origin (especially Chinese),
hypermetropia, female gender and family history. Normal pressure
glaucoma is particularly common in the Japanese population. The central
corneal thickness (CCT) is an important measure to consider when
determining one’s IOP, as thin corneas can give rise to a falsely low IOP,
underestimating the true (much higher) IOP. The average central corneal
thickness is about 540 microns (although it varies on the source you
read…).

11) B
The aqueous is secreted by the non-pigmented epithelium of the ciliary
processes. Aqueous humour is produced via active secretion using a
Na/K ATPase dependent pump (2/3) and passive ultrafiltration through
blood vessels on the anterior surface of the iris (1/3). Aqueous humour
flow is decreased by about a half at night compared to the day, and
peaks again in the morning. About 90% of aqueous is drained by the
trabecular route (which is pressure-sensitive) and only about 10% by the
uveoscleral route. Carbonic anhydrase in found the in the ciliary body
epithelium and drugs inhibiting this enzyme will suppress aqueous
humour production.
12) C
See-saw nystagmus is associated with parasellar tumours (e.g. pituitary
macroadenoma, craniopharyngioma). Downbeat nystagmus is seen with
lesions of the craniocervical junction at the foramen magnum. Upbeat
nystagmus is linked to posterior fossa lesions. Convergence-retraction
nystagmus results from lesions in the pretectal area (e.g. Parinaud
syndrome) or pinealoma.

13) B
A, C and D are all well-known associations with RP. Other associations
include Bardet-Biedl syndrome and abetalipoproteinaemia.

14) D
Advanced Stargardt disease can show bull’s-eye maculopathy. Other
conditions also include Batten disease (lipofuscinosis) and Bardet-Biedl.
A, B and C are all lipid metabolism and storage disorders that can give
rise to a ‘cherry-red spot at the macula’ (a finding most commonly seen
with central retinal artery occlusion).
15) A
The most likely diagnosis is vernal keratoconjunctivitis (VKC). It is an
allergic conjunctivitis that most commonly affects young boys. Clinical
features include Horner-Trantas dots (peri-limbal small white dots) and
giant papillary conjunctivitis (often described as large cobblestone
papillae). Other causes of cobble stoning are atopic keratoconjunctivitis
(AKC) and mechanical tra u ma (e.g. contact lenses). Papillary
conjunctivitis occurs in bacterial infections and allergic conditions. In
contrast, follicular conjunctivitis is seen in viral infections (and some
bacterial exceptions: Chlamydia, Borrelia burgdorferi, Bartonella
henselae) and drug/toxic reactions. Answer C refers to ligneous
conjunctivitis. Preauricular lymphadenopathy occurs in infective
conjunctivitis, most commonly viral.

16) C
Ophthalmia neonatorum is defines as a conjunctivitis occurring within the
first 4 weeks of life. Gonococcal infection has a hyperacute onset (within
1-3 days of birth), with severe purulent discharge. The rest of the
bacterial infections would start on day 4-28 after birth.
17) C
In the UK, all babies born at <32 weeks gestational age or weighing
<1500g at birth should be screened for ROP (definition of prematurity is
<37 weeks gestation). Laser photocoagulation is the most commonly
used treatment option. High concentration of supplemental oxygen is a
known risk factor for developing ROP. Stage 5 classifies the final stage of
ROP and is characterized by total retinal detachment.

18) A
Symblepharon is a partial or complete adhesion of the palpebral
conjunctiva of the eyelid to the bulbar conjunctiva of the eyeball.
Ankyloblepharon is a partial or complete fusion of the superior and
inferior eyelids. Epiblepharon is an eyelid with an extra fold of skin that
causes an in-turning of the eyelashes in the presence of a normal eyelid
position (very common in Asian children). Euryblepharon (B) is a bilateral
symmetrical enlargement of the palpebral fissure.

19)B
PPRF generates horizontal eye movements. Rostral interstitial nucleus is
a part of the medial longitudinal fasciculus and is the vertical gaze center.
Frontal eye fields are primarily responsible for saccadic eye movements.
Superior colliculus is considered the primary integrating center for eye
movements.
20) D
Left INO would result in defective left adduction and ataxic nystagmus of
the right eye on the right gaze. Parinaud syndrome is characterized by lid
retraction in primary position, upgaze palsy, defective convergence,
pupils with light-near dissociation and convergence retraction nystagmus
(pinealoma is a known cause in children). Brown syndrome results from
a mechanical restriction of the superior oblique muscle tendon, when the
affected eye cannot look in and up (elevation in adduction). Duane
syndrome occurs when there is anomalous innervation of the lateral
rectus by the 3rd cranial nerve.

21) A
The Duochrome test uses letters or numbers against the red and green
background. It relies on the principle of chromatic aberration. As red and
green light passes through the lens, they have a different index of
refraction, where green light is refracted more than the red light (i.e. the
green light is focused in front of the red). The test is sensitive to changes
in refraction of 0.25D or less, and is often used to refine the final sphere
in refraction. It can be used in colour blind people.
22) B
During indirect ophthalmoscopy, the examiner remains at a distance from
a patient (c.f. getting close during direct ophthalmoscopy) and used both
of his eyes to look through the indirect ophthalmoscope (i.e. examiner
uses binocular vision, c.f. monocular in direct ophthalmoscopy).
Therefore, the stereoscopic (3D) image of the fundus can be seen with
an indirect ophthalmoscope (c.f. flat 2D image in direct). However,
magnification factor is less (x3 with a 20D lens). In contrast, the direct
ophthalmoscope can magnify the fundus image x15. The field of view
with indirect ophthalmoscopy is much greater compared to direct.
Indirect ophthalmoscopy gives an inverted image, direct ophthalmoscopy
gives an erect image.

23) D
D is correct. Schwalbe’s line is the anatomical line on the interior surface
of the cornea. It represents the termination of Descemet’s membrane
and delineates the outer border of corneal endothelium.
24) C
The boy has tuberous sclerosis (epilepsy, mental retardation, facial
angiofibromas, ungual fibromas; other features include ash leaf marks
and shagreen patches). Astrocytomas (answer C) are closely associated
with tuberous sclerosis. Answer A describes cavernous haemangiomas; B
- capillary haemangiomas (seen in Von Hippel-Lindau); D - choroidal
osteoma.

25) C
The iris findings described in the vignette are called iris roseolae, which
are specific for syphilitic uveitis. Syphilis can affect almost every eye
structure, but anterior uveitis is the most common presentation.
Ophthalmological features related to congenital or acquired syphilis
include: adnexa  gummata, madarosis; anterior segment  papillary
conjunctivitis, interstitial keratitis, roseolae/iris nodules; posterior
segment  choroiditis/chorioretinitis/neuroretinitis/retinal vasculitis;
neuro-ophthalmology  Argyll-Robertson pupil, papilloedema,
retrobulbar/perioptic neuritis, ocular motility disorders, VF defects.
Leprosy (answer B) clinical signs include miosis with iris pearls - small
white spots on iris surface. Learn about ophthalmic manifestations of
Bartonella henselae and Borrelia burgdorferi infections as there
frequently feature in the exam!
26) D
The rods are turned on (depolarized) in the dark and turned off
(hyperpolarized) in the light. Phototransduction cascade in a nutshell: 1)
Retinal and rhodopsin are bound together. Light photons convert 11-cis
retinal to all-trans retinal, making retinal to change shape. This causes
rhodopsin to also change shape, and a part of rhodopsin (called
transducin) breaks off. 2) Free transducin binds to phosphodiesterase
(PDE) and activates it. 3) Active PDE converts cGMP to GMP.
In the dark, cGMP is bound to sodium channels and keeps them open,
and sodium is entering the cell. However, in the light there is less cGMP
(and more GMP), there are fewer sodium channels open, and less
sodium is entering the cell, and the cell is more negative
(hyperpolarized).

27) B
Elschnig spots (small black spots surrounded by yellow halos) and
Siegrist streaks (hyperpigmented flecks arranged linearly along choroidal
vessels) can appear in young adults with accelerated malignant
hypertension (e.g. toxaemia of pregnancy). Copper and silver wiring are
features of chronic hypertensive retinopathy. Other retinal signs of
chronic hypertension include Salu’s sign, Bonnet’s sign and Gunn’s sign.
Flame-shaped haemorrhages, cotton wool spots and papilloedema occur
in severe hypertension
28) A
The patient is most likely has central serous retinopathy (CSR).
Classically, it occurs in young and middle aged men, and stress in a
known risk factor. Typical FA findings include ‘inkblot’ or ‘smokestack’
appearance. Flower petal pattern is often seen in cystoid macular
oedema. Lacy pattern is seen with choroidal neovascularization in
exudative AMD. Hyperfluorescent crests and hypofluorescent troughs
occur in choroidal folds.

29) B
Learn bony anatomy! Roof: frontal and sphenoid; Medial wall: maxilla,
lacrimal, ethmoid and sphenoid; Floor: maxilla, zygomatic and palatine;
Lateral wall: zygomatic and sphenoid.

30) C
Use the RAD-SIN rule to help you out. Recti  ADduction (and oblique
muscles - abduction). Superior muscles  INtorsion = medial rotation
(and inferior muscles - extorsion). Inferior oblique will hence abduct and
extort.
31) C
This is chronic blepharitis (inflammation of the eyelid margins). Classic
symptoms are burning, grittiness, mild photophobia, and
crusting/redness of the lid margins, usually worse in the mornings, and
often with remissions and exacerbations. Vignette describes collarettes
(hard scales and crusts forming cylinders around the lash bases), which
is characteristic of staphylococcal blepharitis. In turn, staphylococcal
blepharitis is associated with atopic dermatitis. Remember that
blepharitis can be subdivided into anterior and posterior blepharitis
(depending on the part of the lid margin affected). Demodex follicularum
LONGUS is associated with anterior blepharitis and demodex follicularum
BREVIS is associated with posterior blepharitis. This table summarises
key features to differentiate chronic blepharitis:
Anterior Posterior
Staphyloccocal Seborrhoeic
Associated Atopic dermatitis Seborrhoeic Acne rosacea
condition derma t itis
Lashes Colarettes (hard Soft greasy No eyelash
deposits), often deposit features
with loss, causing
distortion, or eyelashes to
trichiasis stick
together. May
have loss,
distortion or
trichiasis
Tear film +- dry eye +- dry eye +++ dry eye
+++ foaming
Cornea +- corneal +- corneal +++ corneal
erosions/ erosions/ erosions/
infiltrates infiltrates infiltrates
Cysts Hordeolum/stye None Hordeolum/stye
32) C
This is acute canaliculitis, which is caused by HSV. The canaliculi are tunnels
(one each in the upper and lower eyelids) linking the puncta to the lacrimal
sac. They are initially vertical for 2mm (ampulla part) and then turn
horizontally for 8mm to join the lacrimal sac. Actinomyces israelii is
associated with chronic canaliculitis, and classically causes a pouting
punctum with concretions (sulphur granules that are metabolic by-products
of actinomyces) that can be expressed on canalicular compression. Strep and
staph are more commonly causative for dacryocystitis (lacrimal sac infection).
Remember that acute dacryocystitis is treated with warm compresses and
oral antibiotics (and irrigation and probing should NOT be done). The key
surgery for dacryocystitis is dacryocystorhinostomy (anastomosis between
lacrimal sac and middle nasal meatus).

33) C
Being female is a risk factor for angle closure. The presentation of a red
dilated right eye with severe headache and vomiting should raise the
suspicion of acute angle closure glaucoma. It is typically precipitated by
darkness (as mydriasis closes the angle), hypermetropia, shallow anterior
chamber and is most common is Asians and females.
34) B
The presentation of a middle-aged women with gradual impairment of
central vision is vague. The fundoscopy findings of hypopigmented ovoid
lesions at the posterior pole with cystoid macular oedema point towards
Birdshot chorioretinopathy as the underlying condition in which HLA A29 is
positive in over 95% of patients

35) D
Neck injury whilst skateboarding causing persisting pain and neck stiffness
afterward is consistent with whiplash injury which may precipitate carotid
dissection. Carotid dissection interferes with sympathetic neurons ascending
with the carotid causing Horner’s syndrome hence the anisocoria in darkness.
The structure damaged is the internal carotid artery.

36) D
A dense shield-like anterior subcapsular plaque characteristic of atopic
dermatitis. Approximately 10% of patients with severe atopic dermatitis
develop bilateral cataracts around the ages of 20-40.

37) D
Grave’s disease is the commonest cause of bilateral proptosis and unilateral
proptosis. All of the others are potential causes of proptosis however they
are not as common as thyroid ophthalmopathy related to Grave’s disease.
The major modifiable risk factor is smoking.
38) B
Bull’s eye maculopathy is characteristically associated with chloroquine or
hydroxychloroquine retinal toxicity. Another condition worth noting in relation
to Bull’s eye maculopathy is Stargardt disease which is the most common
macula dystrophy associated with lipofuscin accumulation in the RPE and
mutations in the ABCA4 gene.

39) C
The child has prominent epicanthic folds (particularly common in Asian
children) giving the false appearance of a inward deviated squint i.e.
pseudoesotropia. A Hirschberg test is a simple screening test with a pen
torch. Findings of light cantered around the pupil is consistent with
pseudostrabismus with no true ocular deviation

40) D
Inferior oblique causes extortion as it’s primary action and abduction and
globe elevation as a secondary action. A helpful thing to remember is that
the recti muscles (excluding lateral rectus) tend to adduct the globe whereas
the oblique muscles tend to abduct it as per the RAD SIN rule.
41) C
The finding of a restriction of gaze with compression of the field of gaze on
Hess chart implies a mechanical issue rather than a nerve palsy. This can be
caused by restriction of the extra-ocular muscles by entrapment of fibrosis.
Restriction to abduction and elevation of the eye would be caused by a
restrictive defect of medial rectus and inferior rectus. This is most commonly
associated with fibrosis of medial and inferior rectus associated with thyroid
eye disease. The extraocular muscles that are affected most frequently in
TED in descending order are: inferior rectus, medial rectus, superior rectus,
levator, lateral rectus and oblique muscles.

42) C
This woman is presenting with a triad of ophthalmoplegia, ataxia and
areflexia which is classic for Miller-Fisher syndrome, a variant of Gullain Barre
syndrome. Typically the extraocular muscles are affected first and it can
follow campylobacter infections. Anti-GQ1b antibodies are positive in the
majority of cases.
43) A
The presentation of gradual onset ophthalmoplegia in the first 2 decades is
suggestive of chronic progressive external ophthalmoplegia which occurs in
mitochondrial illness Kearns-Sayre syndrome. Kearns Sayre syndrome is
characterised by CPEO and cardiac conduction abnormalities (hence the
consultant requesting an ECG) and characteristically causes salt and pepper
retinopathy on fundoscopy.

44) D
The OCT image shows an appearance consistent with cystoid macular
oedema. The risk factors for cystoid macular oedema can be remembered
with the mnemonic DEPRIVEN. Diabetes, Epinephrine, Pars planitis, Retinitis
pigmentosa (E), Irvine-Gass syndrome, Venous occlusion, E2-prostaglandin
analogues, Niacin.

45) A
Centrally bulging corneas with scissor reflex on retinoscopy is suggestive of
keratoconus. Down syndrome is a risk factor for this condition. Fleischer rings
(not to be confused with Keyser-Fleischer rings of Wilson’s disease) are
pigmented rings in the peripheral cornea resulting from iron deposition in
keratoconus.
46) A
The axial length of the eye is the distance from the front to the back of the
eye. At birth the axial length is 16mm and in adults the axial length is
approximately 24mm.

47) D
The commonest cause of blindness worldwide is cataract with a prevalence
of 20 million. The commonest cause of visual impairment worldwide is
refractive errors with an estimated prevalence of 153 million

48) C
The presentation of an over 50-year-old woman with unilateral headache
with peri-ocular pain and blurring of vision is suggestive of Giant cell arteritis.
The fundoscopy image shows a chalky white oedematous disc indicating
anterior ischaemic optic neuropathy. The treatment of GCA with anterior
ischaemic neuropathy is steroids.

49) D
The presentation of purulent discharge in the first month of life is suggestive
of neonatal conjunctivitis, the most common organism causing moderate-
severe disease is Chlamydia trachomatis. However, the presence of
hyperpurulent discharge and severe eyelid oedema are signs which point
toward Gonococcus as the aetiology.
50) C
The dioptric power of a lens is equal to the reciprocal of the focal length
measured in metres. 1/0.25 = 4 dioptres.

51) D
The presentation of a fair skinned individual with a pearly nodular lesion on
the lower lid is consistent with a basal cell carcinoma. The characteristic
appearance on histology is peripheral palisading.

52) B
The vitelliform (egg-like) appearance of the macular in childhood is
characteristic of Best disease, the second most common macular dystrophy
after Stargardt disease. Best disease tends to not cause visual impairment in
childhood; visual impairment occurs in middle age. It has an AD inheritance
hence a family history may be noted.

53) A

1 prism dioptre produces 1cm visible displacement at 100cm towards the


prism base.
54) B
He can drive as long as he wears his glasses. The DVLA visual standards for
driving a car are as follows:
a. Must be able to read a number plate from 20 metres (with glasses or
contacts if necessary
b. Acuity of at least 6/12 using both eyes together or one eye if
monocular vision (with glasses or contacts if necessary)
c. Adequate visual field: horizontal field > 120O with > 50O extension
left and right with vertical field > 20O above and below the horizontal
meridian

55) A
This asymptomatic gentleman has a fundoscopy appearance consistent with
angioid streaks. Angioid streaks can be caused by: pseudoxanthoma
elasticum, Ehler’s Danlos syndrome, haemoglobinopathies and Paget disease
of the bone. The back pain with hearing loss and no family history makes
Paget disease the most likely of these causes. Of the investigations serum
ALP with calcium and phosphate are most useful for Paget’s disease.

56) C
The whorled appearance of her cornea is consistent with corneal verticillata
or “vortex keratopathy”. The most common causes of this are amiodarone,
chloroquine and Fabry disease.
57) C
Prostaglandin analogues are the most efficacious treatment for lowering IOP
in primary open angle glaucoma. They can cause several side effects most
commonly conjunctival hyperaemia. Other side effects include eyelash
lengthening and hyperpigmentation of the iris and periocular skin.

58) D
A double ring sign on fundoscopy is characteristic of optic nerve hypoplasia,
a congenital abnormality of the optic disc. It can be linked to maternal
consumption of substances such as alcohol, insulin (hence the relevance of
her Type 1 diabetes history) and anti-convulsants.

59) B
The history of a man hit in the eye with a tennis ball with vertical gaze
restriction is suggestive of a blow-out fracture which most commonly affects
the orbital floor.
60) D
The bones that comprise the medial wall of the orbit can be remembered by
the mnemonic MELS: Maxillary bone, Ethmoid bone, Lacrimal bone and
Sphenoid bone body.

61) D
Heterochromia of the iris has several causes. In an individual over the age of
40 presenting with a cataract the most common is Fuch’s uveitis syndrome.
The nodules on the inner surface of the iris are known as Koeppe nodules, a
sign of granulomatous uveitis. Stellate keratitic precipitates are characteristic
of Fuch’s uveitis.

62) C
The normal thickness of the cornea is between 540 and 560 µm. Anything
below 535 is considered thin and anything above 565 is considered thick.
63) A
This describes medial rectus myositis (inflammation of the muscle). This can
occur in both orbital myositis and thyroid eye disease. However, thyroid eye
disease NEVER involves the muscle tendon, whereas it may or may not be
involved in orbital myositis.

64) C
Fluorescence angiography = bright white vessels due to fluorescein dye
running through. Red-free = white optic disc and lighter grey blood vessels.
Autofluorescence = very black optic disc and blood vessels.

65) D
The majority of eye growth happens in the first 2 years of life. After this, the
eye continues to grow until the late teens.
66) C
Cataract is the leading cause of REVERSIBLE blindness. Trachoma is caused
by recurrent infection with ch la mydia trachomatis A, B, Ba, and C
(remember D-K cause classic chlamydial conjunctivitis), largely found in
areas of poverty, overcrowding, and poor hygiene. The WHO SAFE strategy
stands for Surgery for trichiasis, Antibiotics (single dose azithromycin), Facial
hygiene, and Environmental improvement. Trachoma has an active stage
(characterised by mixed follicular/papillary conjunctivitis) and a cicatricial
stage (buzz words are arlt lines (broad conjunctival scar usually over the
upper tarsal plate) and herbert pits (shallow depressions at the corneal
limbus)).

67) A
Myopes have longer than average eyeballs and rays of light are focussed in
front of the retina. Therefore, they require concave lenses to diverge the rays
of light so that when they do eventually converge to a focus, this happens on
the retina. Toric lenses are required in astigmatism.

68) C
This is acanthamoeba (protozoan) keratitis. In developed countries, it is
frequently associated with contact lens wear, especially if the user washes
them with tap water. Pain is often severe and disproportionate to the clinical
signs. A pathognomic sign is perineural infiltrates (radial keratoneuritis).
Important culture media:
69) B
Keratometry is determination of the curvature (steepest and flattest
meridians) of the anterior corneal surface. Axial length is the antero-posterior
length of the eye. The measurements are plugged into formulae e.g. Hoffer
Q, Haigis, Holladay, or SRK-T, to calculate the IOL power needed to achieve a
given refractive outcome.

70) B
Isolated inherited congenital cataracts also carry a better visual prognosis
that those with co-existing ocular and systemic abnormalities.

71) B
This gentleman has homocystinuria, which is an AR disorder where there is
an accumulation of homocysteine and methionine because there is impaired
metabolism of methionine. Additional features include neurodevelopmental
delay and a marked thrombotic predisposition. Ectopia lentis is almost
universal by 25 years of age in untreated people, and this is inferonasal
typically. Other ocular features include iris atrophy, optic atrophy, cataract,
myopia, and retinal detachment. Ectopia lentis in Marfan’s syndrome is
typically superotemporal.

72) A

The normal visual field extends 90 degrees temporally, 50 degrees superiorly,


60 degrees nasally and 70 degrees inferiorly with a physiological scotoma at
15 degrees temporally.
73) D
Glaucoma and severe ocular injury in general is more likely with alkali burns
than other chemicals, due to the propensity of alkali to penetrate tissues.

74) C
This is pseudo-exfoliation (PXF) syndrome with secondary open angle
glaucoma. PXF involves the deposition of microscopic, flakey, white, dandruff-
like material throughout the anterior chamber. It is visible through the slit-
lamp microscope on various anterior segment structures of the eye, including
the lens capsule, the trabecular meshwork, and the zonules. PXF increases
the risk of glaucoma since it clogs the trabecular meshwork, preventing good
drainage of aqueous. It is also a risk for cataract surgery because the zonules
become more brittle in PXF eyes, increasing the likelihood of posterior
capsule tears or zonule dialyses.
75) A
POHS is a type of fungal uveitis caused by Histoplasma capsulatum infection.
It is usually asymptomatic and the classic triad seen on examination is
multiple white atrophic chorioretinal ‘histo’ spots, peripapillary atrophy, and
the absence of vitritis. Choroidal neovascularisation is a late sign occurring in
<5% of affected eyes. Seronegative arthropathies usually cause anterior
uveitis. CMV retinitis looks like a ‘Margherita pizza’ due to areas of dense
white infiltrations and flame-shaped haemorrhages. It is an important cause
of retinal necrosis. Sarcoidosis typically leads to granulomatous changes,
such as mutton fat keratic precipitates, iris nodules (koeppe/busacca),
‘snowballs’ +- ‘string of pearls’ in the vitreous, and retinal and optic disc
granulomas (seen as yellow/white lesions).
76) D
VKH is an autoimmune disorder involving inflammation of melanocyte-
containing tissues, including the uvea, meninges, and ear. It mainly affects
pigmented individuals, particularly Hispanic and Japanese people. The
prodromal phase involves neurological (mainly meningitis) and auditory
manifestations (tinnitus, vertigo, deafness). The acute uveitis phase can
cause anterior and posterior uveitis. Anterior uveitis is usually bilateral and
granulomatous with Dalen-Fuchs nodules (granulomas situated between
Bruch membrane and the RPE). Note that Dalen-Fuchs nodules are also
found in sympathetic ophthalmitis. The convalescent phase leads to
depigmentation of various tissues causing features such as alopecia, poliosis,
vitilio, and a sunset-glow fundus. These features often persist, but
neurological and auditory manifestations often resolve with time.
77) B
Retinal capillary haemangoma is seen as a round orange-red mass with
dilatation and tortuosity of the supplying artery and draining vein. 50% of
patients with solitary lesions and almost all patients with multiple lesions
have VHL.

78) C
In high myopia, axial length is often >26mm. Maculopathy is the most
common cause of visual loss in high myopes. You can think of the fundus
being ‘overstretched’. As such, the fundus has a pale tessellated appearance
due to attenuation of the RPE with visibility of the choroidal vessels, and
lacquer cracks appear, which are ruptures of the RPE-Bruch membrane-
choriocapillaris complex. Important complications include rhegmatogenous
retinal detachment, macular holes, and CNV.
79) B
Albinism is a group of disorders of melanin synthesis. It can be tyrosinase-
negative (more severe), or tyrosinase-positive. In tyrosinase-negative
albinism, there is no melanin synthesis at all. Visual acuity is usually <6/60,
the iris is translucent (causing a pink-eyed appearance), and there is usually
a pendular and horizontal nystagmus that increases in bright light. The optic
chiasm has MORE crossed fibres than normal, and this can be demonstrated
by visual evoked potentials.

80) C
The spiral of tillaux is an imaginary line joining the insertions (not origins) of
the 4 recti muscles. The medial rectus inserts closest to the corneal limbus at
5.5mm, followed by the inferior rectus (6.5mm), lateral rectus (6.9mm) and
the superior rectus (7.7mm).

81) B
The base of a prism should be pointing towards the direction that you want
the deviated eye to move. Therefore an exotropia (outward pointing eye)
requires a base-in for correction, and an esotropia (inward-pointing eye)
requires a base-out for correction. The tropia is best treated when the power
of the prisms is split between the two eyes (in this case splitting the 6
dioptre correction equally between the two eyes as 3 dioptre prisms).
82) A
< 1 year, forced preferential looking cards e.g. Keeler are most appropriate.
1-2 years, Cardiff acuity cards are suitable.
Kay pictures are used for children with sufficient language to name the
pictures (usually age 2). Sheridan-Gardiner or alternatives such as the
Sonksen (cards each containing a single letter or crowded letters) are used
for children >3 years

83) D
The image formed by a prism is erect, virtual and displaced towards the
apex.

84) A
A bilateral 6th nerve palsy is almost always due to raised intracranial
pressure. The sixth nerve travels through the subarachnoid space where it
ascends the clivus and enters the cavernous sinus. It can be stretched
against the clivus as the brain stem herniates through the foramen magnum
due to increased intracranial pressure.
85) B

The left eye is exotropic and the right eye is fixating, so the right eye will
NOT move when the left eye is covered. Exotropia causes crossed diplopia
(right image comes from left eye and vice versa). Esotropia causes uncrossed
diplopia (right image comes from right eye, etc). Therefore, when covering a
left exotropia, the abnormal image on the right will disappear.

86) C
Exenteration is the removal of the globe together with the soft tissues of the
orbit. It is the best treatment for orbital malignancy, either primary or where
a tumour has invaded the orbit from elsewhere. Enucleation is removal of the
globe with sparing of the soft tissues e.g. muscles. This may be used for
severe trauma, certain primary intraocular malignancies that can be removed
entirely via removal of the globe, or a blind painful or unsightly eye.
Evisceration is removal of the entire contents of the globe with sparing of the
sclera and extraocular muscles. This should never be undertaken in the
presence of suspected intraocular malignancy but can be used for a blind
painful or unsightly eye.
87) C
The central vision requirement is to read a number plate from 20 meters (for
pre-2001 number plates, the distance is 20.5m). For the peripheral vision, a
horizontal field of vision of 120 degrees (at least 50 degrees left and right)
and a vertical field of 40 degrees (at least 20 degrees up and down) with no
defects in the central 20 degrees is required. Stereopsis (3D vision) is not
required for driving. Monocular people can drive provided their only eye has
sufficient central and peripheral vision.

88) B
The breast is the most common primary in females and the bronchus is the
most common primary in males.
89) B
Amblyopia is a poor visual acuity (unilateral or bilateral) where there is no
identifiable pathology in the eye or the visual pathway. It occurs when there
is some form of vision deprivation e.g. due to a cataract and/or abnormal
binocular interaction e.g. in strabismus, during the critical period in a child’s
life when vision is developing. Strabismus is the leading cause of amblyopia.
In strabismic amblyopia, the sensitive period during which acuity can be
improved is usually <7-8 years.

90) C
You can remember this using the OVALE mnemonic: Otic ganglion, V3
(mandibular division of the trigeminal), Accessory meningeal artery, Lesser
petrosal nerve, Emissary veins. It would be wise to learn the cranial fossae,
the foramina, and what passes through them. Commonly tested nerves in the
DE exam include CN II, III, IV, V, VI, and VII. This includes their divisions
and what they supply.
References

Bowling B. (2015). Kanski's Clinical Ophthalmology: A


Systematic Approach. 8th ed. Philadelphia: Saunders.

Elkington AR, Frank HJ, Greaney MJ (1999). Clinical Optics.


3rd ed. Hoboken: Wiley-Blackwell.

Snell RS, Lemp MA (1998). Clinical Anatomy of the Eye. 2nd


ed. Hoboken: Blackwell Science Inc

You might also like